Vous êtes sur la page 1sur 1575
1 © FUNCTIONS AND MODELS 11 Four Ways to Represent a Function Inexorses requiing estimations or approximations, you answers may var gh om the enauas Gian bere, 4.48) The point (~1, ~2) ison the graph of f,s0 f{—1) = ~2. (6) When 2 2.1 is about 28, so f(2) ~ 28, (6) Fla) = 2is equivalent t y (6) Reasonable estimates for when y are When y = 2, we have x = 3 and.x =2.5and.r = 0.3, (€) The domain of {consists ofall 2-values on the graph off. For this function, the domain is —3 0. These values of give real number results for V2, whereas any value off gives areal number result for /F. The domain is 0,00) 30. glu) = V+ VA= Wis defined when w > Oand4—u 20 ¢> w-< 4, Thus, the domain is0 @ <> r(e~5) > 0. Note that 2? Avision by 90. The expression x(x — 5) i postive if < O or x > 6. (See Appendix A for methods for solving ‘nequalities.) Thus, the domsin is (26,0) U (5,20). 2 h(a) = VIFF. Nowy = VI=F > the graph isthe tp half of circle of radius 2 with center at the origin, The domain is fe | 4a? > 0} = {x [42 2°} = (2 | 22 ai} = [2,2]. Fromthe graph, the range is < w S 2,0F (02. Sx £ O-since that Would result in vee gty=4so 38, f(2) = 5 is defined forall real numbers, so the domain is, or (—20, 20). ‘The graph of fis a horizontal Line with y-intercept #6. F(a) (2+) is defined for al real numbers, so the domain sR, ot (20,00): The graph of Fis line with intercept ~B and yinereept 38. {(0) = 2 — bs dened fora zea umes, so the domain is or (00,0). The graph of fis a parabola opening upward since the coetcient of fis postive. To fnd the t-inerepis, let y = Oand solve fort 0=P-Or=4e-6) > t= The t-coordnate ofthe ‘vertex ishalfvay between the intercepts that is at ¢= 3. Since and t = 403) = 9, the verter i (3, 9}, +i . be 36 H(t) = SEE = BADEN sofort #2, HO) =2 +t The domain 4 is {££ 4 2}. So the graph of His the same asthe graph ofthe function hy S(t) = t+ 2a line) except forthe hole at (2,4). 14 (CHAPTER FUNCTIONS AND WODELS {B7. gle) = VF=Bis defined when x ~ 5 > Oor x > 5, so the domains [5, 20). , Siney=VF=5 + y=e-b > ‘op balf of a parabola ¥? +5, we se that g isthe Qetl if ars 20 -Qrtl) iaesied deel fed 2-1 The domain is B, oF (30,20). Set lal 2 ife>o 38. G(e) = ELIE since jx} = tecewene A ee (eso (4 iteve a=) = { 822 eco [E te0 0 (2) =F sinoeiah =)" ig ge sehne 4 ite>o = iteco Note that gis not defined for = 0. The domain is (~c0,0) U (0,00) ala) ape aftt? eso ese fe l= ifz30 ife>2 The domi i hai e42 810) {i ife>-1 Note that for = 1, both + 2 and 2 are equal f, The domain is SECTION 1.1 FOURWAYS TOREPRESENTAFUNCTION 15 249 if e<8 4 fa)=} 22 itis ” a. 5h J. For ~4 3 Note that fr. and ~6 are equal to ~6, Domain is, 3, both 2 +9 and ~2r are equal to 6; and for both ~2 [ow Recall that the slope m of ane between the two points (21,1) and (x2, ya) isn and an equation ofthe Hine 73) connecting those pont isy — yx = mx ~24). The slope of hiss seamen is = "59 — 8,50 an eauation s 2 1). The funetion is f(x} = 10-105 bie Foe pom equation sy 10-= Fh The slope ofthis Hine segment is 5} The function is f(e) = ~32+ §,-9<2 57, ‘We need to solve the given equation fory. © +(y-1)*=0 ¢ (y—1)?= ee y-l=4vz eo = 1 VTE. The expression with the positive radial represents the tp half ofthe parabola and the one with he negative radical represents the bottom half Hence, we want /(2) = 1 — y=, Note tha the domain is 2 <0. a (yH2P ad & HPs das? a y-2a4VTRM yas VTA. Thecop hallicsivendy the function f(z) =2 4 VI=aF, -2< <2. ForO <2 <3, the graph isthe line wit slope ~1 and y-intercept 3, hats, y = —2 + 3. For 3 < 2 <5 the graph isthe line 2%e~B)sory = 2x ~ 6, Sothe fmeton is ‘with slope 2 passing through (3,0); that is, y ~ 0 n243 HoSeca ry ee acres 2), oF y=~Je~3. For-2.< 2 < 2, the graphs the tp half ofthe citcle with center (0,0) and radius 2. An equation ofthe eircle ies? 4 y? <4, son equation ofthe tap halfis y = V7. For 2 < x < A, the aap isthe line with slope $ passing through (2,0); that is,y~0-= $2 —2),0ry 8, So the funetion is fe-3 if -4cr<-2 fa)=\ViT# if 2<202 je-3 if 2ez 0, since lengths must be positive quantities. IP we further restrict 1 to be larger than then L. > 4 would be the domain, 458. Let the length ofa side ofthe egufateral triangle be x. Then by the Pythagorean Theorem, the height y ofthe triangle satisfies V+ (fax)? =a, sothaty? = a? — Js" = 422 and y ~ Yr, Using the formula forthe ca A of atriangle, 4 (base)(height), we obtain A(x) = £(2)(*S.2) = 92%, with domain > 0. E390 L = 9/V, and the surface area is S54, Let the volume ofthe cube be Vand the length of az edge be L. Then V sv) (wy = GV", wich domain V > 0. '55. Let each side ofthe base of the box have length +, and let the height ofthe box bef. Since he volume is 2, we kw that 2 = ha? so that h = 2/ir*, and the surface atea is S = 2” 4 xh. Thus, S{e) = 2° + da(2/2%) = 2? + (8/2), with domain > 0. ® 30 h=30-2 88, The area ofthe window is $a(42)? = 2h +, where his the height ofthe rectangular portion ofthe window. ‘The perimeter is P hye bee fae h= 16020 — x2). Ths, m2) Since the lengths + and hr must be positive quantities, we have 2 > Qand h > 0. For h > 0, we have 2h > 0. > Ae — da? — Sa? 4 get Babee 0s O> 224m re 5. Hence, thedomainof Aisa <2 < Bea tee 57. The height ofthe box is and the length and width are L = 20 ~ 22, W = 12 — 22. Then ¥ = LW and so V (2) = (20 22)(12 — 22)(x) = 4(10 — 2)(6 ~ 2)() = 40(60— 160 +*) = 4x — 64n? + 2408. ‘The sides L, W, and « must be positive. Thus, L>0 < M-2z>0 6 <0; W>0 & 12-2550 © x< b:and.x > 0. Combining these restrictions gives us the domain 0 < 2 < 6, 8. $2.00 if O02 < 1.0 22 iFlLo ei aT 7 foo= Cae So fis an odd funtion. So f isan even function, Since this is neither f(x) nor —f(}, the faneton f is flex) = neither even nor od. ©. f(x) =1 4307 — 24 S(-a) = 14 8(~2)? ~ (-2)* = 14 89? ~ 24 = f(a) So fis an even function 10 f(2) =1 +80" ~ 24,80 Slt) = 143-2)? = (-2)* = 1 431-29) (24) we at Since this is neither J(2) wor ~ f(x), the function fis neither even nor odd 68 Flo) = [2 a) Sle) So f isan odd function. lal = (ele) ‘SECTION’.2 NATHEWATICAL MODELS:ACATALOG OF ESSENTIAL FUNCTIONS Cl 19 4.2. Mathematical Models: A Catalog of Essential Functions 4. @) f(2) = Y@ isa root function with n (©) 2) = T= is am alasbrae fanction because itis 8 r0tof polynomial © h(x) = 2° +2 isa polynomial of degree 9 ra) isa rational function because itis a rai o€ polynomials (©) s(2) ~ tan 2ris aviasnometre function (©) t(2) = fog ba logarithm anton 2. (@)y~ (x ~ 6)/(2-4-6) isa rational function because tis rato o polyoma. (0) v= 2-4 #2/ VET is an algebraic function because involves polynomials and wots of polynomials (011 = 10" isan exponential function oot that ine exponen (@) y = 2"° is.a power function (notice that « is the base). (0) y = 28 + #8 — x isa polynomial of degree 6. (Oy — co + sind is egonometric funtion 3 We notice from the figure that g and hare even functions (symmetric with respect the y-axis) and that fis an od function (oymmeric with spect oe tn $0) [y= °] must be f. Sine slater than near the origin, we must have (© [v = 2%] matched with g and) [y = 2°) matched with A. (a) The graph of y = Bi alne (choice G) ys is an exponential function (choice (©) y= 2 isan odd polynomial function or power funetion (choice F) (8) y = YE = 2° isa r004 funtion (choice 9). 5. (@) An equation for the family of linear funetions with slope 2 is y= S(2) = Be +6 where bis the g-nteroep. 20° CHAPTER FUNCTIONS AND MODELS (©) J(2) = I means thatthe point (2,1) ison the graph of J, We ean use the point-slope form ofa line to aban an equation forthe fail of linear fanetions through the point (2,1). _y— 1 = (sr ~ 2), which is equivalent toy = me + (1 — 2m) inslopeinterept form, () Te belong to both families, an equation must have slope re = 2, so the equation in par (b), becomes y = 22" ~ 3 Is the only funetion that belongs to both faites. 6. All menrves af the family of linear Functions f(x) = 1-4 (r+ 3) have graphs that ae lines passing ehrough the point (—2, 1). 1. ll members ofthe family of linear functions f(r) = ¢~ 2 have graphs ” that are lines with lope —2. The yintercept isc exc 2, 4 The vertex ofthe parabola onthe fet is (3,0), so an equa (a 3)? +0, Since te point (4.2) ison te parabola, we'll substitute 4 for and 2 for yto find a. 2= a(d—3)? = a=2, soan equation is f(x) = 2(x—3)* ‘The y-intercept of the parabola onthe rights (0,1), S0.an equation is wr? 4 br + 1, Since the points (2,2) and (1,-2.5) are on the parabola, we'll substitute —2 for and 2 for y as well as 1 fr x and —2.5 for y to abla two equations ‘with the unknowns a and b fa-%4+1 3 da-%B=1 +41 3 atb=-35 @) 2-@) + (gives usGa= 8 = a= —1, FromQ@),-140=-35 > b= -25,soanequation is ale SECTION 1.2. MATHEMATICAL MODELS: ACATALOG OF ESSENTIAL FUNCTIONS 21 8, Since f(—1) = J(0) = (2) =, f has zeros of ~1, 0, and 2, s0.an equation for f is f(2) = alx ~ (—1)](x— O)(e 2), oF fz) = ax(x + 1)(2 — 2), Because f(1) = 6, we'll substitute I forseand 6 for /(2 6 a()Q(-1) > -2a=6 = 1, So an equation for fis f(z) Sola + 1)(2 2), 10. (a) For 7 = 0.02¢ + 8.50, the slope is 0.2, which means thatthe average surface temperature ofthe world is increasing ata rate of 0.02°C per year, The T-inteeept is 8.50, which represents the average surface temperature in °C in the year 1900. 250°C 2100 ~ 190 200 > 7 =0,02(200) + 8.50 tH. (a) D = 200, so ¢= 0.0417D(a + 1) = 0.0417(200)(a+ 1) = 8.340 +8.34. The slope is 8.34, which represents the ‘change in mg of the dosage fora child for each change of 1 year in age. (©) Fora newbom, a ay (b) The slope of —4 means that fr each increase of 1 dollar fora rental space, the numberof spaces rented deereases by 4. The _yrintercept of 200 isthe number of spaces that would be occupied it there were no charge for each spuce. The 1-interoept of 50 isthe smallest rental fee that results inno spaces rented. 2) (8) The slope of # means that F increases # depres foreach increase of 1°C. (Equivalently, increases by 9 when C inereases by 5 and F decreases by 9 when C' decreases by 6.) The F-inteept of| £32 the Fahrenheit temperature comesponding o a Celsius temperature of “4. (@) Leta = distance waveled (in miles) and ¢ = ime elapsed (in hous). At = §h.d = 40, Thus we 40-0 0 (and at = 50 minutes = 50 aye points: (0,0) and (3,40), 0m = 42=2 — sand so d = a (€) The slopeis 48 and represents the car's speed in mi/h, T.-T, _ 80-7 _ 10 MN; T3118 ~ 60 ee 15. (a) Using in place of » and 7 in place of y, we find the slope to be equation is -80=1(V 173) @ T-80=4N-IB g 7 (by The slope of 4 means that he temperature in Fahrenheit degrees increases one-sixth as rapidly as the number oferckel chimps per minute, Said differently, each ineease of 6 cricket chirps per minute corresponds toan increase of 1°F (©) When = 190, the temperatures ten approximately by 7 = $(150) + 882 = 76.159 = 70°. 2 6. ” 18 8. 2. CHAPTER FUNCTIONS ANO MODELS (@) Let x denote the numberof ehuts produced in one day and y the associated ‘cost. Using the points (100,2200) and (300, 4800), we get the slope ga f2H0 — 2889 = 13, Soy — 2200 13(2 — 100) y= 1S + 900 (b) The slope ofthe linen para is 1 and it epresents the cost (in dollars) ‘of prodcing each additional chai. (©) The y-imercept is 900 and it epresens the fixed daly costs of operating the factory, change in pressure _ 4.34 TO feet change in depth ~ “10 (0,15), we have the slope-intereept form ofthe fine, (@) Weare given 434, Using P for pressure and d for depth withthe point Oagtd +15, iba © 195.85 feet. Thus, the pressure is GP, (6) When P = 100, hen 100 = O.434d +15 <> O4Md = 85 4 100 Ib/in® at depth of approximately 196 fect. _ 460-380 80, S00 480 520 (@) Using din place of and C in place of y, we find the slope tobe ; Soa linear equation is C — 460 = 3 (d— 800) = C~ 460= 3 (b) Leting d = 1500 we get C= 4 (1500) + 260 = 635. The cost of driving 1500 miles is $635. © ‘The slope of the line represents the cost per mile, $0.25 (@) The wincerceptreprosents the fixed cost, $260. (6) A linear function gives a suitable model inthis situation because you have fixed monthly costs such as insurance and car payments, as well as costs that inerease as you drive, such as gasoline, ol and tires, and the cost of these for each additional mile driven is constant (2) The data appear tobe periodic and a sine or cosine function would make the best model, A model ofthe form Jl) = acos(br) + € Seems appropriate (b) The data appear to be decreasing in a linear fashion. A model of the form f(x) = me + b seems appropriate. () The data appear to be nereasing exponentially. A model ofthe form f(x) Wor (2) UF + seems appropriate (©) The data appear to be decreasing similarly tothe values ofthe reciprocal function. A model ofthe form f(x) = a/- seems appropriate. ‘SECTION .2 MATHEMATICAL MODELS: ACATALOGOF ESSENTIAL FUNCTIONS 23 ‘Some values ar given to many decimal laces. These ar th resus given by several compute algebra systems — rounding is et tothe reader, AO 5 () Using the points (4000, 14.1) and (60000, 8.2). we obtain 82-14 = AL (2 4000) oes Y= hd = GEG (© ~ 4000) oF, equivalently, 1y ® 020001053572 + 14.5214. 1 ° 6100 ‘A linear model does seem appropriate (6) Using a computing device, we obtain the least squares regression line y = ~0.0000097855. + 13.950764, ‘The following commands and screens illustrate how to find the least squares regression line on a TI-83 Plus. Enter the data into list one (L1) an list two (L2). Press [STATI] to enter the editor. 7 Ee a0 Find the regession line and store it in Y. Press [2nd] [QUIT] [STAT] fb] [4 [VARS] [9] [7] EN aby Foca TeReReey Vim] CInReS_ Fare oa S:37a546e-5 | [Passe -Sx+i3, 3807) Siabb7edos | |é4arro8s "Note from the last igure that the repression line has been stored in Ys and that Plot has been turned on (Plot is highlighted), You can tur on Plot from the Y= menu by placing the cursor on Plott and pressing [ENTER] or by pressing [2nd| STAT PLOT] [1|ENTER] pe ee Buse, lssBTotsore Now press ZEON) to prodose «graph of the dat andthe repression ‘i line, Note that choice 9 of the 200M menu ntomatcally selects a window ‘that displays all ofthe data (@) When x = 25,000, y * 11.456; or about 11.5 per 100 population, (e) When 2 = 80,000, y ~ 5.968; or about «6% chance (£) When 2 = 200,000, y is negative, so the model does not apply 24 OGHAPTER1 FUNCTIONS AND MODELS 22. (@) 220 enn ose) (©) When 2 = 100°R, y = 264.7 ~ 265 chinps/min, Be) 2@ wg 20 yea ‘A linear model does seem appropriate (©) When x (a) When 2 100, o 2704chigsnin 4s ash) Using a computing device, we obtain the last squares regression line y = 4.85% ~ 220.98. © 2006 gt 2000 oe Using a computing device, we obtain the least squares regression line y = 0.0801197472 ~ 1582408249, where 2 i the year and yi the height in feet. 2000, the model gives y= 20.00 Rt. Note thatthe actual wining height for che 2000 Olympics is less than the ‘winning height for 1996—so much for that prediction. 28,91 ft This would be an increase of 9.49 ft from 1996 to 2100, Eventhough there was an increase (08.59 8 from 1900 101996, it is unlikely tata similar increase will occur aver the next 100 years, 24, By looking atthe scatter plot ofthe data we rule out — the linear and logarithmic models. We ty various models: Seater pn nn) Quadratic: y = 0.496? — 62,289Rr + 1970.69, Cubic: y= 0,02012432012" — 8.880372062° + 247.675446 Rr — 5163.985208 Quanic: y= 0,0002951049r* ~ 0.06845600052° + 5.275256412? — 180.226051 Le + 2208.210056 Exponential: y = 241422904 (1.054516914)" Power: = 0,00002285.407123 01078291 uwiatic moist RS (eset) ei mae SECTIONS NEWFUNCTIONSFROMOLOFUNCTIONS O25 ey cone a cosas After examining the graphs of these models, see that the cubie and | quartic models are clearly the Best aporentiateniet “AHO! Powermader “anton 2. 6200 ion) Using a computing device, we obtain the cubic function y = ar* + hx? + er + d with 2 = 0.002837, 6 = ~7.06142, ¢= 12,823, and d = ~1,743,770, When 2 = 1925, 9 1914 (million) wa 2010 ea %. (a) T= Logan garg! ye 750 (b) The power model in part (a) is approximately T Squaring both sides gives us‘T? = dso the model matches Kepler's Thi Law, T? = ke’, 1.3_ New Functions from Old Functions, 4. (a) Ifthe grapt of fis shitte 8 nits upward its equation becomes y = f(z) +3 (b) LF the graph of fis sited 3 units downward, its equation becomes y = f(z) ~ 8 (6) Ifthe gragh of fis shifted 3 units tothe right, its equation becomes y = fla ~ 3). (4) Ifthe graph of fis shifted 3 unit tothe lef, its equation becomes y = f(x +3) (6) lithe graph of fis elected about the 2-axs its equation becomes y = —/(2) (ff the graph of fis reeted about the yrans, its equation becomes y= f—2) () I the graph off is stretched vertically by a factor of 3, its equation becomes y = 3/2). (i) Ifthe graph of J is shrank venially by a istor oF 3, its equation becomes y = 4 (2) 2. (a) To obtain the graph of y = 5,f(2) from the graph of y = f(x}, stretch the graph vertically by a factor of 5. 4) Tobtin the grap of y = fe ~ 5) fom the graph of y = f(), sift the graph 5 unis the igh, (6) To obiain the graph of y = ~ f(x) fom the raph of y = (2) reflect the raph about he z-axis (@) To obtain the graph of y = —5(x) ftom the graph of y = f(x), stretch the graph vertically by a facto of 5 and reflec it about the 2-axis, () To obtain the graph of y = f(6:) from the graph of y = f(x), shrink the graph horizontally by a factor of 5 (0) Toba the graph of y 3 units downward fle) ~ 3 from the graph of y = f(x), stretch the graph vertically by a factor of 5 and shift t 2 GRAPTER’ FUNCTIONS AND MODELS 3. (@) (graph 3) The graph of fis shifted 4 units to the right and has equation y = f(x ~ 4) () (graph 1) The graph off s shifted 3 units upward and has equation y = f(e) + 3. (6) (graph 4) The graph off is shrunk vertically by a factor of 8 and has equation y = 4 f(x). (2) (graph 5) Te graph of fis shifted 4 units tothe left and reflected about the axis, Its equation is y Sle +4), (6) (graph 2) The graph of fis shifted 6 units othe left and stretched vertically by a factor of 2. Its equation is, y= 2fle+6), 4. (@) To graphy = flr + 4) we shift she graph off. 4 units to the let “The paint (2,1) aa the aragh of f corresponds tothe point (2 ~ 4, 1) =(~2,1), () To graph y = 2f (3) we strech the graph of vertically by a actor of 2 ‘The point (2,1) on the graph of F corresponds tothe point (2,2. 1) = (2,2) 5. (@) Tograph y = f(2r) we shrink the graph of £ horizontally by a factor of 2 ‘The point (4, —1) on the geaph off corresponds tothe point (4,1) = (2,-1. (6) To graph y = f(2) + Awe shift the graph of f units upward, ‘The point (2,1) on the graph of F corresponds tothe point (2,1 +4) = (2,5), (2) To praph y = —4/(x) +3, we shrink the raph of f vertically by factor of, then reflect the resulting ‘graph about the 2-axis, then shift the resting graph units upward ~~ a % ‘The point (2,3) om the graph oF J corresponds tthe point (2, —$ 148) = (2.2.5). 4b) Toaraphy = f(z) we stretch the sraph of f horizontally by & factor of 2 ‘The point (4, ~1) on the graph of F eostesponds tothe point (24,1) = (8,1) SECTIONS NEWFUNCTIONS FROMOLDFLNCTIONS CI 27 (6) To graph y = f(—2) we reflect the geaph of f about (@) To.graph y = —f(—2) we reflect the graph off about the yaxis the y-axis, then about the e-axis ‘The pont (4,1) on the graph of f corresponds tothe point (-1-4,—1) = (4,1). The pons (4, point (—1-4, 1) onthe graph of F corresponds to the 1-1) = (41) 6. The graph of y = f(2) = Ve 2 has been shifted 2 units to the right and stretched vertically by a factor oF 2 Thus, funetion describing the graph is = 2f(2 2) = 2D Br 6— (Fe F) = VP FTE 1. The graph of y = f(x) = VE —2? has boen shifted units tothe let elected about the 2-axis, and shifted downward ‘Luni. Thus, a fianetion describing the graph is y= te feta = 1 reflect shift shin aboutr-axis 4 units eT unit left ‘This funetion canbe written as, y= —Sle+4)—1 =~ VSS DFP — 1 = — Se FD FF) ~ 1 = Ve 8. (a) The graph of y = 2sin-r can be obtained from the graph (0) The graph of y = 1 4 y% can be obtained from of y = sin x by stretching it vertically by a factor of 2 the graph of y = by shifting it upward 1 unt, a 8. y = ie Start withthe graph of y ~ = and reflect. about the a-axis, Note: Reflecting about the yaxin ives the same esl since substuing ~2 for gives 28 OCHAPTER1 FUNCTIONS AND MODELS. “+1: Sur withthe graph of y = 22, reflect about the x-axis, and then shift 1 unit upward. 4t. y = (e+ 1)% Start with the graph of y and shift unit tothe lef Dyaxt dr 5H (ete ha) and then shift 1 unit downward, 3} yewn2eot 13. y= 1+ 2008 Star withthe graph of y = cos. stretch vertically by a factor of 2, and then shift unit upward, pecan yo 2eosx yodoose tt 44, y = Asin32: Start withthe graph of y = sin, compress horizontally by a factor of 3, and then stretch vertically by a factor of 4. yesiny posing 7 yedsinge ot AAA SY SECTION.3 NEWFUNCTIONSFRON OLDFUNCTIONS Cl 29 48, y = sin(x/2): Sta withthe raph of y 2 and stretch horizontally by a factor of 2 16 y = 3/(e~ A): Start withthe graph of y = 1/ and shift units to the right VFB: Start withthe graph of VE and shift 3 units to the lef 8. y = (2-4 2)* + 8 Star withthe sreph of y yeetes | 18. y= Sa +80) = He? + 8+ 16) 42-44)? — 8 Sta with dhe graph of y = x, compress vertically by & ‘actor oF 2 sift 4 units othe left, and then shift 8 units downward. vey = > yohea? yoheap—s 30 Cl CHAPTER? FUNCTIONS aND MODELS y= 1+ YEAH: Stare with the grap of YF, shift unit tothe eight, and then shift 1 unit upward, eke | = | am : 2/(a + 1): Start withthe graph of y = 1/2, shift 1 unit tthe Ie, and then stretch vertically by a factor of 2 Nes wa NQesh \ tance, shift units tothe right, and then compress vertically by a factor oF 4. Stan(a — y=tanie— 4) 24. y = |x? — Be] = Jz? —2e +11) = |(e 1)? ~ 3: Sta withthe graph of y = 2, shift t unix ight, shift unit downward, and reflect the portion of the graph below the 2-axis about the x-axis. SECTION.3. NEWFUNCTIONS FROM OLOFUNCTIONS <3 25. This is just lke the solution to Example 4 except the amplitude ofthe curve (the 30°N curve in Figure 9 on June 21) is 14 ~ 12 = 2 So the function is £(¢) 2+ 2sin{355(¢ ~ 80). March 31 isthe oth day of he year, 0 the model gives ‘1(90) & 12.34, The daylight time (5:51 AM to 6:18 PX) is 12 hours and 27 minutes, oF 12.45 h. The model valve differs from the actual value by 248-224 «0,000, less than 1% 28. Using a sine finetion to model the brightness of Delta Cepel asa function of time, we take is period to be 5.4 day, its amplitude tobe 0.5 (onthe scale of maaitude), and its average magnitude to be 4.0. Ifwe take t= O ata time of average brightness, then the magnitude (brightness) asa function of time € in days can be modeled by the formula M(t) = 4.0 + 0.85sin( 25) B. (@) To obtain y = f(z). the portion ofthe graph of y = /(2) tothe right ofthe y-axis is reflected about he y-axis, (6) y = sin v= Va 28. The most important features ofthe given graph are the 2-interepts and the maximurn and minimum points. The graph of y = 1 /f() bas vertical asymptoes atthe e-values here there are 2-intercepts onthe graph of y = J(2). Tke maximum of 1 on the graph of y = fe) comesponds ta minimam of 1/1 = Lon y = 1 /f(e). Sima, the ‘minimum oa the graph of y = f(2) corresponds to a maximuns on the graph of y=1/S(e). As the values of y get large (positively or negatively) on the graph of y= J(2)the values of y getclose w zer0 on the graph of y = 1/2). 2. fx) = 2" 422%; ofa) = 32" D=R forboth and 9. (+ a)a) = (2 + 22%) + (Be? 1) = 29 +52? 1, D=B. (f= a)(e) = (2 + 224) ~ (a2? 1) <2" 28 +1, D=R. (o)(e) = (a8 + 202)(30? — 1) = 30° + 624 — 2 25%, D=R, (2) = S228. v= feet ty) mea —140 0, fle) = VIRB, D = (00,3); gfe) = VEF=T, D = (~20,-1]U 1,00). (4 ge) = VI=E + VERT, D = (—00,~1]U 1,3), which is the intersection of the domains of f and g (F - ge) = VITE- VAT, D = (v0, 1.3 (oe) = VIRB. VAT, D= (00, —U,3) (S»- £ 20) 1) U (1,8), We mist exclude r = 2 since these values would make £ undefined. 32 CHAPTER’ FUNCTIONS AND MODELS a. slo D=R ofe)= (@) (fog)(2) = flgte)) = e+ ©) (92 Sle) = a(F(2)) = we? = ¥ (fo fa) (f(x)) = Aa? ~ (x' —2r? + 1)-1=2'- 22, D=R. (@) (9 09)(2) = aloe) = 922 + 1) = 208 +1) 41 = (4242) +1 = 42+3, D=R fle) = 2-2 ole (42° 4 4e +1) —1= 42" + de, D=R Qs? 2) 412s 1, D=R 274244. D=Rforboth f and g, and hence for their composites. (@) (F2.)(2) = S(ole)) = fa? + 8x + 4) = (at 4+ Be +4) —2= 97 +2 +2, () (90 A)le) = al fle) = ola -2) = (@— 2)? + 80-2) + 444432 -644= (©) (f° F(a) = SF(@)) = Fle 2) = (@-2)-2 4 () (g09)(2) = 9lg(2)) = ole? + 82+ 4) = (22 +80 44) +804 + Be +4) 4 A400? 416-4 62% 4 82? 4 2dr) 43x" + 90+ 1244 460! +202? +930 +92 3B. f(z) = 1 ~ B25 g(2) =ensx, D=R forboth f and g, and hence for their composites. (g(z)) = f(cos.x) = 1 — 3e08:r, (0) (fo F(@) = FF) = f(t ~ Se) = 1 ~ 31 ~ 32) = (8) (g0.a)(z) = 9(a(2)) = a(eos2) 84 0r= 00 —2, eos(cos2) [Note that thsi no oes - 6082] 4M f(a) = VE, D= (0,00); gle) = YT=%, D=R. @) (Fo9)(@) = fale) = FYB) = VT 8 = mae, The domain of fogs (r| YT=F > 0 0) (90 (2) = aS(@) = (VE) = Y= VE ‘The domain ofg © fis {2 | xis nthe domain of {and f(x) isin the domain of g}. Tiss the domain off, tat, (0,00) (Fo Nie (@) (goatz) {2|2 <1} = (201 (LE) = F(VE) = VTE = YE. The domain of fo fis {x | x > 0 and YF > 0} = (0,00). a(o(z)) = 9( YT =z) = YT VT =a, and the domain is (00, 20) 6 sle)=att, D= (2240); g(a) = 225, D= (ele 4 2} (© (Foane)= sae) = s(Z45) = 45 42) “reo re _ GANG +) +(r42e42) _ (42 t+ HAe td) oot Hors - FGF) rer er VerD —2 and f(o()) isnot defined for = —2 and «= |e 4 -2,-1). Since g(2) isnot defined for 2 the domain of (f © )(2) is 1 1 2 z oot) =a) =9(r+4) fede pe 2a ‘Since f(z) is not defined for 2 = 0 and g(f(2)) isnot defined for 2 = the domain of (go f)(z) is D = (e| ¢ # ~1,0). © ene=s0e)=4(2+ 2) = = Rea +1) +1 fe? +1) + ale) _ teeta tide? a +1) er wi43r741 — aera) Pa ele40 E+I+ ie +2) =o(244 ete atl tet? _ Bet +2 Since (2) isnot defined for x = 2 and (g(x) 8 not defined for = the domain of (90 9)(2) is D = {| 2 # -2,—$}. fle) = TE D={2| 24a} ole) = sind, D=R. (@) (f 0 9)(2) = fla(x)) = f(sin22) = sine Tr sind Domi: sn 2240 entry te eX SS 42m oe 248 bam (rninee) © ae.Nta) = ate) =o( 25) =sn(2E). Domain: 2 4 2) a =) -+e) c(rente) = 5000) = 1785) = me [ -rets Lo = Te (6 (9 9N(9) = gla(e)) = g(sin2x) = sin(2sin2x). Domain: 3. (Fog oh)(x) = f(olh(e))) = flaw — 1)) = f(2@ - 1}) = Ae—1) +1 =2e-1 8. (Fog oh)(x) = Flo(h(e))) = flg(d ~2)) = (1 2)*) = 21 — 2)? ~1 = 2a? ae +1 ®. (Sogoh}(x) = lalh(a))) = flale" +2) = fla" +2)*] = No? +40 +4) = VEE D3 = VT (A)- (5) 41, Let oa) = 24 + Land f(2) = 2°. Then (f © 9)(2) = flale)) = fa +1) = (22 + 1)! = F(a), 4. (Fog oh\(x) = Slalh(a))) = lol YB) = 42. Let (2) = Vand f(z) = sin. Then (f 0 9)(z) = flgl2)) =F (VE =sin(y#) = Fl). 4. Let (2) = YF and f(2) = -T>. Then (F ea)(e) = fala) = HR) = aor 44, Let g(x) = p25 and f(e) = YE. Then (f o9}(2) = Sloe) = = Gla, 1% CHAPTER FUNCTIONS ANO MODELS 45, Let g(t) = cost and f(t) = v2. Then (F 0 9)(t) = S(a(t)) = f(eost) = Veost = u(t) tant T+tant 48, Legit) tantand (0) = 5. Then (Fo9)(t) = flat) = Fant) = a) 41. Let h(x) = 2°, (2) = 3, and f(z) (Fog oh(2) = stolle))) = flole*)) = #(3") = 2. Then 8 Let h(x) = [2 oz) = 2-+ 2 and f(z) = 7% Then (Fegany(a) = Sla(&(a))) = fale) = £24 le) = TFT = HE) 49, Let h(x) = V, g(x) = secx, and f(x) = 2°. Then (5 899A\G2) = flolh(a))) = flaly%)) = Hoe VE) = (oe o)* = se! (ye) = Ha). 50. a) (9) = (0) = 5 ©) gista)) = 98) =2 (©) ff) = £8) = @) a{9(1}) = 91 (€)(90 f\(9) = 91F¢0)) = 98) = © (F296) = f19(6)) = 113) =4 81. (a) (2) = 5, because the pont (2,5) ison the graph of g, Thus, f(g(2}) = (5) = 4 because the point (5,4) ison the raph off. ©) 9(F(0)) = 910) © (F09)(0) = H(9(0)) = £3) =0 (4) (9 £)(6) = a(F(6)) ~ o(6). This va is vot defined, because there is no point onthe graph of g that has coordinate 6. (61 (90.9)(~2) = aa )FoNa) = FA) 3) = 91) = 4 (2) = 2 S52, To finda particular value of f(a(2)). say for 2 = 0, we note fom the graph that (0) © 2.8 and f(2.8) = ~0.5. Thus, F(9(0)) & F(28) ~ -0.5. The other valoes Viste i the table were obtained in a similar Fashion. = | ate) | Fle) =| oe) [ Fo) 5 | 02 | —4 o| 28] 05 -4| 12] -33 v} a2} a] 22] —17 2] 12] ss -2} 28] -05 3] 02] -4 -1} 3 | -02 4| 19] -22 sf m1] 9 53. (a) Using the relationship distance = rate rime with the radius ras the distance, we have r(2) = 60, () A=ar® > (Aor)(t) = Aleit) = x(600)? = 360072, This formula gives us the extent ofthe rippled area (acm?) at anytime t ‘SECTION1.3. NEWFUNCTIONS FROM OLDFUNGTIONS O35 54, (a) The radius ofthe balloon is increasing ata rate of 2 cm/s, so r(¢) = (2 em/s){Us) = 2 Gem, (6) Using V = $4, we get (V 0 r}(t) = V(r(®)) = V(t) The result, V (208 = Bat “Zr, pies the volume ofthe vllvon (ia ca") a a function of time i). 455, (o) From the igure, we have a right triangle with legs 6 and «and hypotenuse 5 [By the Pythagorean Theorem, d? +6? = a(t) = 3508 {@) There is « Pythagorean relationship involving the legs with lengths d and 1 and the hypotenuse with lengths: @ 41 =F Thus, old) = VFL ©) (80d(t) = s(d{t)) = (350t) = y(3508)7 +7 5.0) 1 © e 0 irtco 0 iftco - vi so V(e) = 12001 HO {' me © {to frog HO o 4 Stating withthe fom in pa (owe enlace 120 wit 240 to reflect the sof different voltage. Also, because we restating 5 units tothe right of = 0, wwe replace ¢ with ¢— 5. Thus, the formula is V(e) = 240%1(¢ ~ 5), 0 eco ° it RW) =H) OVO=1) ocr OVO= Man were 0 ifto s0 V(t) = 2H(t), €< 60, so Vit) = {4 TMU — 7). t 32, ¥ y nat ” Ore oF 7 58. F fe) = mys + by and glee) = maz + ba, then (£og\(e) = S(gla)) = formas + ba) = mi (mae + ba) + by = mma + maby +b So f 9 isa linear function with slope mime, 36 CHAPTER FUNCTIONS ANDNIODELS 8.16 AGe) = Le, then (Ae A)(2) = A(AG@)) = A(L.042) = 1.040.042) = (1.08)%2, (Ae Ao Alle) = AG AS A\(a)) = AQ(LON)?2) = L04(1.04)*2 = (1.04)"x, and (Ao Ao AoAl(x) = A((Ae Ae A)(z)) = A((L.04)*2) = 1.04(1.04)8z, = (1.01) “Toese compositions represen she amount of the iwestment after 2.3, and years. Based on this pater, when we compose n copies of A, we get the formula (A A.o--0-A)(x) = (1.04) 64. (a) By examining the variable terms ing and A, we deduce that we must square gto get the terms 42° and 4 in h, Ie Tet fle) = 2? +e then (fo 9)(2) = flgie)) = f(2r+1) = (2e +1)? tem As? + Ae + (1 +6). Since (x) = de? + de +7, we must have 1 +.¢= 7, $0.0 Gand f(x) = 27 +6, (0) We need a function 9 0 that ((g(z)) = B{al2)) +5 = h(x). But Nx) = 822 + Se +2= (2? +2) +2= (a? + 2 ~ 1) +5,s0we see that g(x) = 27 +21, {2 We needa function g so that g(f(2)) = gl + A) = ha) = Ax —1 = A(x +4) ~ {T. Sowe see thatthe fimetion g must be g(t) = de = 17 63. (a) If f and g are even functions, then f(—s) = f(x) and 9(—2) = g(z). @ (fF +9)(-2) = f(—2) + 9(-2) = f(z) + glx) = (f + 9)(2), 90 F + g is an even function. Gi) aha) = (a) oh) = $8) ale) = (Fa), 80 Fg is an even fanetion, (0) and g ro ftons, then (#2) = f(r) am g( 2) ~ ~af) WF + 9-2) = f-2) + g(a) f(x) + [-9(2)] = —[f(2) + 9(2)] = -F + 9)@), $0 f +9 san od finetion, Gi) (f9)(—) = f(—2) - g(-2) = —F(2) - [-a@)] = f(@) g(a) = (f9)(2), 80 Fis an even function. 64. IF fis even and g is odd then f(—2) = f(x) and g(—2) = 9(2). Now Lah) = F(a) 9-2) = F(2)-[-9(2)] = ~1/(2) -af2)] = ~(f9)(@), 50 F9 isan odd function 65, We need to examine i(—z). Wow) = (Fo a\(-2) = f(al-2)) = F(a(2)) (because giseven) = h(x) Because h(—2) = h(x), is an even function, 66, h(—2) = f(o(—2)) = f(—a(e)). Atti point, we can’ simplify the expeession so we might try to find a counterexample 10 show that his not an odd function. Let g() = ¢, at odd funetion, and f(2) = 2? + 2. Then h(z) = 2° +2, which is neither ‘even nor od. Now suppose J is an odd function. Then f(—g(x)) = ~flgl)) = ~h(x). Hence, h(—2) both f and g are odd. — (4), and so is odd it Now suppose f isan even function, Then f(—a(sr)) = f(a(}) = h(a). Hence, h(x) = h(a), and Soh is even ig is ‘odd and / is even. SECTIONS GREPHINGCALCUATORS AND. COMPUTERS C37 4.4 Graphing Calculators and Computers 1 fle) = Vi (9) [-5,5]by 3] 6) (0.10) by jo,2) (There is no grap shot) ES LET LZ ‘The most appropriate graph is produced in viewing rectangle (c) ) [0,10] by (0,10) = 162? +20 () [3,3] by [3,3] 2 | 6) [10,10] by 10,10 (©) {50,50} by (- “The mas appropriate graph is produced in viewing rectangle fd. 3. Since the graph of f(r) = 5 + 202 ~ 2° js parabola 4. An appropriate viewing reeangle for ‘opening downward, an appropriate viewing rectangle f(z) = 2" + 802 + 200 should include the high and low pois should inelude the maximum point. = 38 CHAPTER FUNCTIONS AND MODELS §. fle) = TF isdefned when 81 ~ 24 20 oe 6 fle) = VOTE TMs defined when Oz +2020 8,0 the domain of (8,5. > ~200, 0 the domain of 6 (~A0, 20), sleet o ef iso ¢ VETS GST = 9,00 te ranges (3, 1, The graph of f(2) = 2* — 2252 is symmettc with respect the origin. Since f(a) = 1 ~ 225r = (2 — 295) = a(x + 18) ~ 15) there are intercepts a0, ~16, and 15, j(20) = 3500. & The graph of fl2) = 2/(e* + 100) is symmetric with espect tthe origin, 8. The por of yl} ~ sin( 20006) is rp 0.0058 is ace LU a) Sino fe) = sin{1000r } is the square of 9, its range is {0.2} anda viewing eagle of [002,401 by 01.2] sem axpropriae 10, The poried of (2) = cas(0.0012) is 5357 = 6300 and its range 1s [-1, lj, soa viewing rectangle of 10,000, 10.006) by 5, 1.5} seems appropriate 1A, The domain ofy = Vis x > 0,50 the domsin of f(x) =sin Ys (0,90) and she range is [1]. With a inl Witand-enor experimentation, we ind {hax an Ne of 10 ilstaes the goeral hape Ff 50s appropriate Viewing sectngle is [0 100) by {2.5.3.5 SECTION 14 GRAPHING CALCULATORS AND COMPUTERS 39 12, One period of y = sees oecuts onthe interval (~$,) U (48) $< Me <> << Heorequvalemly, 0.025 < 2 < 0075 18, The ister, 20sin.2; fas period 2 and range [10 10), Iwill be the dsinant tn in any “arg” 1y = Wsin.r + sin 10h, as shown inthe fist Figue, The second term, sin 1008, bas period 255 = 3 and range [~1 1) causes the bumps in the fis figure and will be the dominant term in any “small!” graph, as shavsn in the view near the origin in the second figure 18. y= 22 40.02 5in(508) 18, Wemust solve the given equation fry to aban equation for the upper and lower halves ofthe ellipse det e214 ay 1 * 4 : 11. From he graph of 0.282 ~ 2.26 in the viewing rectangle [1,3] by [-25, 1.5), its sificult1o se ithe graphs intersect. in i Le and Le 200m non the fourth quadrant, wwe se the raps donot intersect. 40 CHAPTER? FUNCTIONS AND MODELS 18, From the graph of y = 6 — 42 ~ 2 and y = Se-+ 18 inthe viewing. 2 rectangle [6,2 by [5.20], we see that the graphs intersect twice. The points of intersection are (-~4, 6) and (~3, 49, From the graph of f(2} = 2° ats ofthe equation (2) = O and its slighty larger than 9 By 2aoming in oF a wing a oot or eo feature, we obtain == 9.05, ~ Oa — 4, we see that there one sofution 20. We soe that the graphs of f(e) = 2 and glx) = 4a — 1 interseet tinge times, The coordinates ofthese points (which ae the solutions of the equation) are approximately ~2.11, 0:25, and 1.86. Altenatively, we could > = Sind these values by finding the zeros of h(x) = 2" — Ae +1. 21. We se tha the eraps of fc} = 27 and gz) = sin. intersect twige. One solution {2 ~ 0, The other solution of f = 9 isthe a-cootdinate ofthe point of inverseston inthe fst quadrent, Using an inersect feature or zooming i, iw fad his value 1 be approwionanely 0.8, Alternatively, we ‘ould Find that vate hy ining the positive zero oF hz) = 2? ~ sina ote: Ser prcing the graph on 2 T1-83 Plus, we can find the approximate value 0.88 by using the following keystokes: (Zod EALOB ENTERIENTER) 1 ENTER), rhe 1s justo guess or 88 22. (a) 1s The e-coordiates ofthe three pointy of insersestion are C7 Post 23nd (sn wt on fil at 88, Na a ma ese 28, gla) = 58/10 is args than fle) 2, fle) = 2 ~ 1000 is yar than ofa) <2? whenever > 100, whenever > 101. send os 200 Jig SECTION14 GRAPHING CALCULATORS AND COMPUTERS <4t 25 ots ‘We see few the graphs of y = |sin2'— x and y = 0.1 that there are 10 soltions tothe equation sin a ~ 2] = 0.1; = ~0.85 and + 50.85. ‘The condition [sin — 2] < 0.1 holds for any tying between these two values thats, 0.85 < a < 0.85, 28, Plz) = ar! ~ 50" + 4x, Q(x) = Bx". These graphs ae significantly different only inthe egion close to the ovigin. The larger a viewing cectangle one chooses, the more similar the wo graphs look, 2 op om “aaa 21, (a) The ro0t functions y = (0) The root functions y (6) The root functions y = Vy = VF, ve and y y= Yeand y= 9% y= YF and {@) # Forsany n, the nth root of 0s 0 and the mth rot of 1is 1; thai all mh oot factions passthrough the points (0,0) and (1,1) + For odd, the domi of the nth rot function is, while for even n, tis (» € R | 2 > 0) «Graphs of even r00t funetions look similar to that of V/% while those of 9 oot funetions resemble that of YE: «Asm inceases the araph of / becomes steeper nearOand ater for > 1 28, (a) The fametions Aeand (©) The funetions y = 1/2 and (6) The functions y = 1-7. y ve, yet vast y= 1/2 and y = i/o = (8) © The graphs ofall functions of the form yy = 2ye® passthrough the point (1). «# If mis even, the graph ofthe Funetion i eniely above the z-axis. The graphs of 1/2" for n even are similar to one another, ‘IF nis odd, the function is positive for positive x and negative for negative 2. The graphs of 1/0" form aud ate similar tw one another. ‘© As increases, the graphs of 1/2 approach 0 faster as 2 ~* oe, 42 ChAPTERT FUNCTIONS AND MODELS 28, J(v) S04 +22 4, We < —1.5, ther are three humps: ¢wo minimum points anda maximum point. These humps get Mater as ciereases, uni at ¢= 1.5 0 of the humps disappear and there is only one minimum point, This ingle rump then moves 1o she tight and approaches the origin ase increases, 3. fle) = IEE. te < 0, ee tunetion is onty defined on {-1/V=E.1/ V7 [gant graph is the top tof an clips We graphs the tine y = 1. Ire > 0, the graph is the top alo typectole. Ase , 0 pproaches 0 hese curves become later and approaet elie = 2 CaS os By S22, Asm increases, dhe maximum oF the “ oo foneion moves farther fu theo, and gets lage. Noe, towers tu are of he function approaches 38-00 (AS, 4), yo th. he tee become tetera increases =e" +27, We <0, the loop isto the right ofthe origin, and ites postive, its 10 he te In bos eases, the eloser cis tO the larger the loop is, (In the fisting e282, 0~ 0, she Joop is “infinite tha sit doesn't lose.) Also, the larger jt is, the steeper dhe slope is 09 she Toopless side of the oigin 4. (a) y= sin VF) (Oy = sale) This fnetion isnot periods i oscillates fess ‘This fumeciam ascites more Srequently as i} increases. frequently as increases Note afso dat eis fetion is even, wheneas sin. is odd, °| 00 ost 5 SECTIONS EXFONENTILFUNCTIONS 43 38. The graphing window is 95 pixels wide and wo want to stat with = = G and end with x = 2x. Sine there are 94 “gaps ooven piel, te gitance Between pels is 257%, Thus, heals that the calculator ately plots are» Where n = 0, 1,2,-.., 93,94, Por y= sin2r, the actual points plotted by the calculsor are (25 -n,sin(2 BF -n)) for a Pi, the points plated are (34+ n,sin(96~ 36 -)) for = 0,1... 4, But 0,1... 94. Fory in(2nn +2 32) sin(00- 8 -n) sin(O4- 25-422 n) in(2-25 0 (YR (©) (0,0) _(@) See Figures Le). &O), and 6(a), respectively 2. (a) The mumber es the value of such thatthe slope ofthe tangent fine at x =O onthe graph of y =" is exactly (eX 2.71828 (e) Fo) 3. Allofthese graphs approach 0 as > ~2o, all of them pass throuel the poins (0,1), and all of them are inereasing and apyvoseh oo as —+ oo. The larger the base the faster the finetion increases for > O, andthe faster it approaches Das 24-0, Note: The notation “x — 00" can be thought of as "x hecomes large” at this point “More details on ths notation are given in Chapter 2 4. The graph of eis the reflection of the grap of e* about the y-axis, and the raph of 8 isthe reflection ofthat of 8” about the y-axis. The graph of 8 ‘increases more quickly than that of for > 0, and approaches 0 faster 44 CHAPTER FUNCTIONS anD MODELS 8. The finetions with hases greater than 1 ("and 10") ae increasing, while hose ov ses les har 2 {(4)" and (5) *] ane decreasing. The graph of (49° isthe refletion of that of* about the y-ais, and the graph of (4) is the selection of that of 10 shout the y-axis, The graph of 10" ineseases more quickly than that of F fore > O.ant approaches 0 fster as 2 + —00. {Each of the graphs approaches oo as — ~2o, and eal approaches as 4+. The smaller the Bose, the Fister he Funetion grows 8-2 ~ —00, and the faster preaches 0 as 2 —> 86 7, We stat with the graph of y (Figure 3) ane then if 3 units downward. This shift does’ act the omni, but he range of y =a — 38 (3,00) ‘There isa horizontal asymptote of y 18, We-start with the graph of y = 4 (Figure 3) and then hill 3units tothe right. There isa horizon asymptote of y = 0, 9, We start withthe graph of y = 2* (Figure 2), refley taboun the y-axis, anche about the :2-axis (or just rotate 180° to handle both reflections) obtain the raph of y = 2°" Ineach graph, y = is the horizontat asymptote ” ¥ 40, We sact withthe graph of y = e* (Figure 13), venically » stretch bya factor of 2, and shen shit # ae upwear. There is A horizontal asymptote of y = 1. | ye Gh veil syst" y: SECTIONS EXPONENTAL FUNCTIONS 45 “41, Westar with tho graph of y = «* (Figure 13) and reflect about the yas to get the rap of y = e°*, Then we compress the raph vertically by a factor of 2 to obtain the graph of y ang then reflect about the x-axis to get the graph of | Je-*. Finally, we shift the graph upward one unit to get the graph of i u 12, We sae withthe graph of y = o* (Figure 13) and reflect about the 2-axis to get the graph of y = ~e*. Then shif the graph ‘upward one unit to get the graph of y = 1 ~ e*. Finally, we stretch the graph vericaly by a factor of 2 vo obtain the raph of y= 201-6") yerd-e'l 43, () To find the equation of tho graph that results from shifting the raph of y = e* 2 units downward, we subteat 2 from the ‘original function to get y = e* =2 (8) To find tho equation ofthe graph that results from shifting the graph of y ~ e* 2 units to the right, we eplace x with 2 — 2 in the original function o wet y = e°—®? {6) To find the equation ofthe graph that results fRom reflecting the raph of y — e* about the x-axis, we multiply she original function by —1 10 get y = =e" (2) To fi the equation ofthe graph that rests from reflecting the raph of y = e* about the y-axis, we replace with ain she original function to wet y = {) To find the equation of the graph that results from reflecting the raph oF y = e* about the x-axis an then about the axis, we fist multiply the original function by 1 ((o get y = ~e") and then replace «with —x in this equation to geyser 14. (a) This reflection consists of frst rellectng the raph about the «axis (giving the graph with equation y ~ —e") and then fing this araph 2 4 = 8 units upward, So the equation is y = —e* +8. (b) This reflection consists of first electing he graph about the y-axis (giving the graph with equation y =e") and then shifting this graph 2 2 — 4 units to the riht. So the equations y = e° 15. (a) The denominator 1 +e is never equal 10 zee because e* > 0, s0the domain of f(=) = 1/(3 +e") is. I/(1 — eis (—20,0) 4 (0,00), (Ie 0 ee =1 49 2 = 0,50 the domain of f(a) 16. (a) The sine and exponential functions have domain R, so g(t) ~ sin(e~") also has domain R. (6) The function g(t) = VT=F has domain {t |1—2" 2 0} = {| 2" <1} = ft] 4 <0} = (20,0). 48 CHAPTER FUNCTIONS AND MODELS ‘i 17, Use y = Ca withthe points (1,6) and (3,24). 6=Cat [C=] and29=Ce? = w= (Sot * = a=2 [sincea> 0} and “The fnetionis f(z) = 3-2 28, Given the y-iercep (0,2), we ave y = Ca” = 2a*. Using the point (2,3) givesus $= 20? + faa > (3) [sinee« > 0). The fetion i (2) = 2(4)* oe $=) Sek) ~ Ke ih $8 IF fLe) = 5% then «(&) 29, Suppose the month is Fedmuary. Your payment onthe 28th day would be 22°-! = 2° = 134,217,728 eens, or $1,842,177 28. Clearly, the second method of payment results ina farger amount for any month, a2 2A in, J(24) = 248 in = 87 in 4B A. g(24) = 2 jn = 2 /(22. 5280} i= 265 mi 2. We see ftor the graphs that for: less than about 1.8, (2) = 5° > f(x) = 2°, and ther mear the point (1.8, 17.1) the curves intersect. Thom f(x} > 9() fom 2 © 1.8 until = 5, AL(5,3125) there is another point of intersection, and for 2 > S we see that (2) > F(2). In oot, 9 increases much more rapidly than J Geyond that pot psy ye 3 : } ~ = ea) ” », 7 24, We graph y ~ e* and y = 1,000,000,000 and determine where momen % 10°, This seoms tobe tue at x ~ 20.723, 500" > 1% 109 foe > 2 SECTION 15 EXPONENTALFUNGTIONS a7 2, (a) Fileen hours represents 5 doubling periods (one dating period is three hours). 100 2° = 3200 (b) In hours, there will be ¢/3 doubling periods. The initial population is 100, 0 the population ya time tis y = 100-2". conn (t= > y= 100-29 = 10,159 (€) We raph ni = 100-2" and ys = 50,000. The two curves intersect at .£ © 26.9, 50 the population reaches 60,000 in about 26.9 hours, Q * 28. (a) Three hours represents 6 doubling periods (one doubling perio i 30 minutes). 500-2" (©) int hours, there will be 2¢ doubling periods. The initia population is 500, so the population y at time tis y = 800-22" oe) (t= BaF > y= 500-29 ~ 1260 (@) We graph ys = 500. 2% and yx = 106,000. The two curves intersect a 1 3.82, so the population reaches 100,000 in about 3.82 hours. a s 27, Anexponential model is y = ab! where 2 = 8.151832560 x 10-'* one ion and b = 1017764706, This model gives y(1993) © 5498 million and ‘y(2010) = 7417 millon woo 200 28. Anexponential model is y ~ all, where a = 1, 9976760197589 x 10-* andi 1 = 1.0129394821607. This model gives y(1925) © 111 milion, (2010) = 330 million, and (2020) © 975 milion rob 200 rom the graph, it appears that fis an odd function (f is undefined for = 0). fle). ‘To prove this, we must show that /(— 1 Toei = aed s0 f isan odd function 48 CHAPTER’ FUNCTIONS AND MaDELS phe fora = 0.1, 1, and, 29, We'llstrt with b= 1 and graph f(a) = eh From the graph, we see that there isa horizontal asymptote y = Das. —» =o the intereeptis (0,4). and a horizontal asymptote y = Las. oo. Ifa ‘As a gets smaller (loge to 0), the graph of / moves left, As a gets larger, the graph of f moves right, ‘As b changes from —1 © 0, the graph off is streched horizontally. As b ‘changes through large negative values, the graph of fis compressed horizontally. (This takes care oF negatives vafues of) 1b is positive the yraph off is reflected through the y-axis Last, i = 0, the graph of fis the horizontal line y = 1/(1 + a. 1.6 Inverse Functions and Logarithms 4. (a) See Definition 1 (b) Ih must pass the Horizontal Line Test. of fis Band the range of >" is A 2 & Fla) Fle) # Sar), s0 J isoneto-one, fh gla) = 1s. ty #22 > Mer Ae + g(a) #9 (62), s0gis one-to-one. Geometric solution: The graph of g isthe hyperbola shown in Figure 14 jn Section 1.2. Kt passes the Horizontal Line ‘Test, 50 9s one-to-one, (22), $09 isnot oneto-one 18. A football wi tain every height h up tits masimam height wie: once onthe way up, and again onthe way down, Thus, even if doesnot equal fa Fla) may equal ft, 90 fis no 448 not 2 bocause eventually we all stop growing and therefore her are two times at which we have the same height. 45. Since /(2) = 9and fi 1, we know tha f-"(8) = 2. Remember, ihe point (2,0) ison the praph of, then the point e 16. (a) First, we must determine 2 such that f(x) = 8, By inspection, we see that ifr =D, then F(x) ~ 3. Since fis 1-1 (f isan increasing fumetion), ina an inverse, and f~" (3) = 9. fs on the praph off! (b) By the second cancellation equation in (4), we have f(J-*(5)) = 5. ‘7, Fitst, we must determine « such that a(x) = 4. By inspection, we sce tat iC ~ 0, then g(x) = 4. Since g is IL (a isan increasing function), thas an inverse, and g™!(A) = 0. 48. (a) Js 1-1 because it passes the Horizsmtal Line Test. (6) Domain of f = [-3,3] ~ Range of f-*. Range of f {©) Since (0) =2, F"2) (6) Since f{-1.7) 0, f-40) = 1. 1,3} = Domain of f-! 418, Wesolie C= (PF —32) for C= F-92 > F= 0 +82. This gives usa fornia fr the inverse funtion, hat is, the Fahrenheit temperature F asa funetion of the Celsius temperature 459.67 = 7% 491.67 + C 2 ~ 273.15, the domain of the inverse function wee( 8) = one This Formate eves us che speed oF the paricle in terms oF mass tha is, = f~" (on). Use)- VOR + ya VO W29 > F Iocerctange x andy: y Oar Bea? > zoe Bot 2B. Sof Ma) = “+ AL Note that the domain of J FH om warts) eae-1 3 apt gyede 2p ay tt ade ary Bra on utt BEE 5 p-1qq — EET yt t= Que > 2 = POSE merchange andy y = FTE, 80 fe) = SE y= fa) = = n= <> ea Un, imerchange x and y: y= Ving. So f-"(2) = Vinw. 50 CHAPTER FUNCTIONS AND MGDELS oftas Interchange eand’ys y= {25 3.80 JF ~ 3, Interchange x and ys y = a yaork eat = ay yretina) @ arte iz) [Note thatthe ange of f and the domain of {~"is (0, § Rye seysxel = y- Ses YTHT (not + sinoe VEHT, $0 f(r) graph oF y = UT is just he graph of y ~ 4/F shifted sight one unit. > 0), Interchange « andy: rom he weap, we see that fan f~* ae reflections about the Hine 3 = =y 2 Inl2~y). Imerchange andy: y = In(2~ 2), $0 J-"2) ‘hat J and J" are reflections about the line y = 2 12 2). From the graph, we ste 28. Reflet the graph off about the Tine y = x. The points (~1,~2), (I), (2,2), and (3,3) on f ae reflected to (~-2,-1), (1.1), (2,2), and (3.3) oa fo! 30, Reflect the grap of f about the line y = 2 31, (a) It is defined as the inverse ofthe exponential function with base a, thats, log, z= y +> a = (8) (0,00) oR (6) See Figure 11 SECTION 1.6 NVERSE FUNCTIONS AND LOGAATHNS 32. (a) The natural logarithm isthe Jngarth with base e, denoted In x (©) The common logarithm is the logarithm with base 10, denoted log x (©) See Figure 13 5 = since 58 = 12 logy de = ~S since 3? = 4 = 3, @) logy (25 = Bsince 5* = 125, (0) logs Jp = —Bsince 3? = = 3 3. (@)In(1/e) = nt ~ in a (0) losyg WTO = fog 10° = 4 by ( 36.) lon 6 ~ logy 15 + logy 20 = toga (+ logs 20 [by Law 2) = loge +20) [by Law 1) = log, 8, and log, 8 = 3 since 2° = 8 () Logs 100 — logy 18 — log, 50-= logs (32) — logy 50 = low (885) = log,($), and log, (3) = -2since 3 Het a (hE (630 (oe) ng) 19 3. In5-+5n3=In5+In3% [by Law3} = In(5-3°) by Law 3) =Ini215 8. Infa +8) + In(a—b) 2 inf(a+bMa—H Inc Poy Lawes 1,3} eee (by Law2} or nF 38, In 42°) +4 Ina ~ nin = In(1 + 2°) + In.2!/? — nsine = In{(1 + 2°)V) — Insing = bo 6. orga 10 =H x osm (tg 84-= BE a sore 41 Tosnphtese ints ewe a2 = BE nfs = 2 “These graphs all approach ~co as ~+ *, and they all pass through the point (1,0). Also, they ate all inereasing, and all approseh 20 as +o. “The fmetions with larger bases increase extremely slowly, and the ones with smaller bases do so somewhat more quickly. The finetions with large bases appeacle the ais more closely as 2 = 0% 42, We see thatthe graph of Ins the reflection ofthe graph of e* about the line y = x, and hat the grap of fog +i the reflection of the graph of 10° bout the same fine, The graph of 10° inereases more quickly than that ‘of e*, Also note that logo 2 ~» 90 a8 2 ~» oo more slowly thai. Gree 3 CHAPTER FUNCTIONS AND MODELS 20 9 = 68,719,476,736. In miles, this is 48. 38 = 36 in, so we need x such that loge = 96 Ga 1h Lei in DR Nath 994.9877 mi 88719476.796 in EE RE me LOSS. i 84 > g(x} = tne for approximately {2} foe Fromm the graphs, we 9 that fe} 3.06 < 2 < 9.49 3 20" tapprosinately). AC stat pointe eraph off Goalty surpasses the graph of g for good. 5) Reflect the graph oy = fn about be x-axis obtain the groph oF y = ~ Ine 45. a) Shin abe graph of y = logya 2 Ave ues to the Beta logyg(e +8). Note the venial ‘bin the graph oF asymptote of = =5, 7 7 ( | 5 a7 7 at 3 Y= ogy9 2 y= logge +5) yews ins 4, (@) Reflect the raph oy = In-about the yavistoobiin—(Q) Reflect he potion a the graph of y = bn. 0 the ight ‘of the yraxis about the yroxis. The graph of y= In} In(-2). the graph of is that reflection in addition to the original potion 4 47. (@) 2m wer 48. Ge — () Inf — 22) SESS lom 8 | in3 oo reba e BOP © lon On BF =3 ee INQ) =m3 & (@-S)n2—mn3 @ 2~ 18260. The quadratic formula (with a (Ine +In(z— 1) =In(e(e—1)) = ale) =e! 6 = tang (14 VET), but we reject the negative oot singe the natural logsrithe isnot defined for 2 <0, So. e) gives 2 H+ VTE) ‘SECTION 1.6 IWVERSE FUNCTIONS ANDLOGARMINS C3 we Mae @ cs =InCt+be & 50. (a)in(inay <1 eM) =e! ge Ine Ce tne = infe(e)] & ax InCibetine™ o a noo (o-deane o 2= BS ye" abe St. (er <10 > Ine r—1 Se > 0 ape! + rE lI/e,00) Bw@rcmrcd + Pedcd + coed + zee) yet > 4 > Ine >Ind > 2-Be>Ind S —Be>Ind—2 > 2 <—Hlnd—2) we (-on, H(2—In4)) 53. (a) For (2) = VOT PF, wemusthne3—e* 2G eM <3 Be Magy? > Bats) > (by y= FQ) VIE PF fote thaty > 0) > = JS), Interchange x andy: y = $ln(3 — #2). So fx) = $ in — =), For the domain of J, we must haved—a?>0 3 acd > [f 0, Note thatthe domain of f-!, (0, V8), equals te range off. 54. (9) For fz) = In(2-+ In}, we must have? + Ine > 0 = Ing > 2 > a> e-*, Thus. the domain of f ste O)y=fe)=In(2+inz) + oF Binz > nzsel—2 > e=el”* tmerchange x and yy =e So f-*(2) = e°9 The domain of 4 8 well asthe ange OF fi 85, We see that the graph ofy = (2) = V7 Fa F HTT is increasing, so Fi 1-1 $ Ewer: = Y@F FF PF Land use your CAS to solve the equation fo g Using Derive, we got two (relevant solutions involving imaginary expressions, 48 well as one which can be simplified to the following = Ja) = 8 (VDT DB - (DEAF H+ YB, 2 where D = 3 V3 VOT = Wat FT, = Maple and Mathematica each give two complex expressions ang one real expression, and the real expression is equivalent 12) 82M to that given by Detive, For example, Maple’ expression simplifies to $F —2 M = tose? + 12 VIBTTHET TAT — 00, £8, (0) we use Deve then solving 2 = y+ for y gives ws six solutions ofthe form y = 8 VBT=T, where te asin 4 son(4-+2) ,-tom( 448) hott an (2522) (2 Misy= A VI=Twith 8 = asia $ + 5) Where “The inverse For y ht because the domain of A is (0, 2], this expression is only valid for € [0 54 CHAPTER FUNCTIONS AND MODELS Happily, Maple gives is the ret of the sofutiont We solve x = y+ y* fory to ge the two rea saktions Mi YORORAIST @ ore ED pene © = 108 + 12 VB VETTE =A), and the inverse fory ral 2 isthe postive sofution, whose domain is (34.98) “Mathematica alsa gives two real solutions, equivalent 1 those of Maple, © ‘The postive one is 8 (yap 5 + 2Y8D- Ws where Da ~24270 + BVIVEVTEH A. Although this expression also has domain (i.00), Mathematica is mysteriously abe to plot the solution forall > 0. nat = Bags B.G)ma tin. 2! > Bena nl: = £ + toes (8), ving foul 10, we can wt In(ru/200) sys gunction tells us bow tong it wil take to obtain n bacteria (given the umber). thisast = Mn) = 9. use (n= 50,00 > Fon = BCE (EP) m 26.9 nous tle “ue 88.10) Q=Qo(1—e"l*) = 2 a & > -tew(-2) = ‘aln(1 ~ Q/Qo). This gives us the time + necessary to obtain a given charge Q. (Q=09Qvanda=2 = t= —BIn(1 ~0.9{Qo/Qe)) = —2In01 = 4.6 scons 2 (opnin-*(8) § since sin § = Mf and § isin (4.4) (b) c05" 4-1) = w nce aos = —2 and isn 0, x) 6, (a) tan? (5) = f since tan j = by and § isin (3,4). (0) woo" 2 = § since see $ = 2and § isin [0, $) Ue, 4), Gt. (@) acctant since tan f= 3 and isin () sin"! oy = § sincesin syand F isin [-3,4 62. (a) cot”?(~vB) = 4 sinew cot 32 = VB and isin (0, x). (©) arecos(—4) 2% since cos % = 4 and & ini 0, 65, (a) In generat, canfaretan:z} =o forany teat number. Thus, taerfacctaze 20) = (0) sin“ (sin) = sin (sin $) = sin 9G = § since sin § = and isin [-$, 3] {Recall hat & 2» and the sine function is pesiie with period 2x.) SECTION.6 IWVERSEFUNGTIONS ANDLOGARITINS Tl 55 54 (4) Le18 denote the angle whose soeant is 4, that i, 8 = sec“ 4, a8 shown in the figure Thus, tan(sec™4) = tand = FE = VIB af We Alternative solution: sePO=14tan@ > a1 4tantO + tand= V5 (Wotethatsec@=4>0 = Gisin{0,$) + tand>O) T (0) Let = sir-4(Z) [ove the iguel, 5 sin(2sin*(3)) = sin(20) = 2 sind cod = 2(8)¢2 65 boty = sin, Then 3 Sy SH > cosy > 0,s0.00(s 66. Lety = sin“? 2, Then sin y =, so fom the triangle we see that tan(sin“!e) = tany 67. Let y = tan“ x, Then tanty =, so From de triangle we see that Vee sinftan”!2) ~ sing = 68. Lety = tan" x, Then tany = 2 so ftom the triangle we see that ‘cos(2 tan x) = cos 2y = C06? y ~ sin? y (ate) - (aay ‘The graph ofsin“ ws the reflection ofthe graph of Sinz about the line y = x yosins yosin tc “F 5 CHAPTER’ FUNCTIONS AND MODELS n. Eytan The graph of tan" x isthe reflection ofthe graph of = sin" (Be +2). 1 §eeso}s Domain (9) = (x(t <8e-+3 <2} = fx) -2 684 <0) = fe) ~ 72. (0) f(a) = sin sina) Since one function undoes sat the opher ane does, we wet the Identity fetion,y = x, on the restricted domain —2 <2 < 2 (©) ala) = sin-“(an) “Thin is simi vo par (a, bt with domain {Equations fr g on imervals ofthe fem (6 #70, + an), ar any imegern, ean be found wsing lx} = (2) # (2). “The sine function is monotonic on each ofthese intervals, and hence yi g (atin a Hiner toshion) 73. (a) Ifthe poiat (1p) ison the graph of y ~ f(a), then te pont (x ~ cy) i ha pot sified units 0 abet. Since F 8 1) and the point corresponding to (2 — 6, onthe graph of ff 14, the poine (y,2) isan the graph of y = (ye =») on the graph of fF, Thus, the eurve’s reesion is sied bmn the some numberof units asthe curve itself is Ha) <6. shilled to the lef. So an expression forthe inverse function is g°! (2) = (©) It we compres (or stretch) a curve horizontally, the cure's election in the ine y = «is compressed (or stretched) vertically by the same factor Using this geometric principle, we see that the inverse OF A(x) = fer} can be expressed as fe) = eh Mea CHAPTER’ Review O57 1 Review CONCEPT CHECK 4. (a) A funetion f is a rule that assigns to each clement 2 ina set A exactly ane element, called /(x), in a set B. The set Ais cal the domain ofthe futon. The range off the se of al possible ales of fe) a. ais throughout the domain (b) JE f is @ Renetion with domain A, then its graph is the set of ordered pairs {(x, f(x) | x € A}. (.) Use the Vr Line Test on page 6 2. The our says fps anton a: vray, numerical, vnuly, and algebra, An example ofeach ven below Verbally: An assignment of sues chars na sesso sein in won) [Numerically tx tabetha signs an mount fax oan income (a table of vals) Visually: A graphical history ofthe Dow Jones average (a graphy _Akebracalys A rtonsipBtweon distance, rasan tine: = anil) 3. (@) Aneven function f satisfies f(x) ~ f(2) for every number in its domain, It is symmetei with respest to the jeans (&) Aw ode function g satisfies g(x) = .a(2) for every number in its domain, I is symmetic with respec tothe origi, 4. A function fis called increasing on an interval if fxs) < f(a) whenever 2 < 23 in J 5. A mathematical model isa mathematical description (often by means of function or an equation) ofa real-world pPhenomenci, 6. (9) Linear Runctions f(r) = 20-41, fle (8) Power Funetion: f(x) = 2%, f(a) = 2 {c) Exponential function: (2) = 2°, fle) =a* (6) Quadratic Function: (2) = 2? 42-41, (2) = a2" bbe be arth (6) Polynomial of degree 5: f(a) = 2° 2 (1) Rational function: f(x) = 25, fe) = FES where P(x) ana We) Q(x) are polynomials 8) o) $C GHABTER} FUNCTIONS ANDIODELS o o @ © @ w 8, (2) The domain of f +9 is the intersection of the domain of f and the domain of g that i, A.0.B. (8) The domain of fy is also AB. (©) The domain of f/'g must exclude values of» that make 9 equal to 0; that, (2 € AB | gtx) # OF 10. Given two functions Fund 9, the composite function fo gis detined by (f © 9) (x) = f(g (x)}. The domain oF fog i the set ofall x inthe domain of g such that g(x) i in the domain of f 1 (a) Ifthe graph off is shifted 2 units upward its equation bevomes y = f(x) +2, (b) Irthe graph of fis shifted 2 units d (6) tthe grap of f i shifted 2 units tothe righ, es equation Becomes 3 (4) AF the graph off i shifted 2 units tothe Left, is equation becomes y = fl +2) Se). (0 Lethe graph of fis reflected abpat the y-anis, its equation becomes y = f(~). nward, is equation becomes y = f(x) — 2 (e) IF the graph off is reflected about the x-axis, its equation becomes y = (6) Ifthe graph of fis stretched vertically by @ factor of 2, its equation becomes y = 2(2 {h) Ifthe graph of J i shrunk vertically hy a factor oF 2, its equation becomes y = Ef («). (0 tr the graph of Fis stetched horizontally bya factor of 2, is equation becomes y = J3)z G) tte grap of fis shrunk hormonally bya factor of 2 ts eaustion comes y= f{22). 1 (a) A faeton Fiscal amet faction i it never takes on the same vale ce: hai fms) # Flea) whenever 201 #2. (Ox f is 1-1 ifeach output eoresponds to only one input) Use the Horizontal Line Test: A funetion is one-to-one if and only if 0 horizontal ie iaterseets its graph more than (b)1Ff fs one-to-one function with domain A and range B, then its imerse function J has domain B and range A and is defined by fa) {or ary y in B. The graph of f-* is obtained by reflecting the graph off about the fie g 2 6 fle) » CHAPTER REVIEW O99 18, (a) The inverse sine fusion f=} = sine is defined as follows: sintesy & sinysa and Ndmain tet So ¢tantisnmes-Z sys (6) The inverse cosine finetion f(r] = cos! sis defined as allows: corey + cosyae amd Oy flea ‘Soe the Vertical Line ‘est Lt f(2) = 28 and gle) = 2x. Then (f2g)(e) = fol) > f(22) = (2e)* = a0? and (92 Na} > oJ) = fe") = 22". 80 Fog # aof. Lot f(x) = 2°, Then Fis one-to-one and f2) = 9/F. But 1/f(2) = 1.2%, which is not equal to J~"(3) ‘We can divide by 6° sinee &* # 0 for every. “The function hn i an increasing function on (0,00). Let Lote =e? anda = ibe, Wht, weve tht In 1, bue since te range of tar" as (— fis true thar tan 3 = we must have any”* (3) = — 2. For example, tan” 20 is defined: sin“! 20 and ens? 20 are not 6 OCRAPTER’ FUNCTIONS AND MODELS: EXERCISES 4. (ah When 2 = 2,y = 2.7, Thus, (2) = 27. Wsej=3 > 2s 23,56 (6) The domain of f is ~6 << 6, 0r|-6,6) (8) The range of fis -4< y BexL > exL D=(~o0,4)U(f.00) Range: all eats except 0 (y = 0 ie the horizanal asymptote for f.) R= (36,00) U (0.30) 6. g(a) = VIER Domain: 16-2820 2 aS > BIS VB > [fsa D=}-22) Range: y>Oandys VIR > ASy sa. R={6,4) TNa)=In{2+8). Domains 246>0 = 2>-6 B= (8,00) Range: 2-4 6 > 0,s0n(2-4 6) takes onl eal numbers and, hence, the ranges B= (20,08) Domain; B, ~00,00) Range: —LScos2t s2>9 > ol (2) 3 > FE (-00,~3)U (00) ©) (oo Ne) i(Inz) = {Inz)? 9, Domain: 2 > 0, or (0,00) CHAPTER REVEW O83 (© (Fo fx) = fUE(@)) = f{nz) = In{Inz). Domain; nx >0 = 2 > €° = 1, or (1,00) jix? 9) ~ (2® — 9) 9. Domain: x € B, 08 (00,08) 0, Let hx) = 2+ YE a(t) = VEand f(x) = If, Then (Po go h(x) = bee = Pla) ate) Ja) = ge) = aang = Fa, @ (goal) = alae) ‘Many models appear to be plausible. Your chaice depends on whether you a ® ‘ink medical advances will kep increasing life expectancy, or if there is ‘bound to be w natural leveling-off of ie expectancy. A linear model, (0.2403 — 423.4818, gives us an estimate of 77.6 years for the v0 ao year 2010, 22, (a) Let x denote the numberof toaster ovens produced in one week and ‘y the associated cost, Using the points (1000, 9000) and (1500, 12,000), we get an equation of line: 990 = 2.000 = 9000 5.999) = 9000 = 22000 = 9000 fe — 2000) y= 6(z— 1000) +9000 > y in + 8000, | Sw i og (ease ven) (©) The slope of 6 means that each atonal toaster oven produced adds 86 to the weckly production cst {6) The y-iteroep of 300 represents the overhead eost—the cost incurred without producing anything. 2, We need o know te value of 2 Such that J) = 2+ Ine = 2 Since = 1 gives usy = 2, JQ) = 1 PEE ary tesytl 2 tyoyalar ait 24 y= ZEAE Inerchanging x and y gives us 2 = WF Imerchanging x and y gi wat yr) = 25. (a) 2 = =Peo 20g (25-2) = log 100 = Fogg 10? = 2 (8) logy 25-4 ony 4 (6) tan(aresin }) = tan} = 4 Thensin(oos"? 4) =sind = vIn = 1-8) =f (@) Let = cas" $, 50.0050 = (Der =F > (yInz=2 > 2 ain? > es In(in2) we 158m) (4) tar > tantan’ (4 CHAPTER) FUNCTIONS ANDMODELS 27) 900 ‘The population would reach 900 in about 4.4 yeas » > OOP + 900Pe°* = 100,000 9n0Pe 100) a y(_OF . (EP) a5 Bp) ti etn 100,000 ~ 100 => 000 100P __, _, (100—P) 007" oe) requived for the poputation co reach given number P. 9.900 709 — 900, P= + rm 5) = nye ap. 109 te 7 Fox large values of x, y = a has the largest y-values and y they ae inverses of each other. (PRINCIPLES OF PROBLEM SOLVING By sing the area formula fora range, § (base) (height), in wo ways, we se that BO (W) = $A) (@),s0.0= 2. Since? + y? = hy = VIER TE, and VRE 2 Refer to Example 1, where we obtained k= 2 =! the 100 came from oP “4 times the area ofthe triangle, In tis ease dhe aea ofthe triangle is P? = 406) iP 4 (R)(12) = 64, Thus, = > 2Ph=Pt2ah = 2Phe2h= PF = AEP LBW)=PP > syeeaf22 824 ga pee ftt® Fees Be Lom peed TOS nao tec ‘Therefore, we comderthe tre cases 2 < 5, ~B << hands > Wee wemust have 1 = 2r (2-5) =3 4 r= 3, which i false, since we ave considering 2 < ~/ Ws <2 < f,wemusthnve] 22 5)39 te > }, ye mst have <(e+d)=3 & 2=9, So the no sotutions ofthe equation are ar = rol feel e-3 ita es abs and [2 — 3) (eee ipret Bx ites and x = 9. Therefore, we consider the three cases 2 < 1,1 <2 << S,andin > 3, Wa < Lwenust ave 1—7— (8-2) 25 0D 7, which is fe. i <3, wemust have x—1- (3-2) 25 <> 92 ¥, which false because x <3, Wx 2 B,wemusthaver—1~(x—8) 25 22 5, which ise All toe cases ead to falsehood, so the inequality has no soation, 5. fle)= Case (> 10 <0 < 1,then f(a) = 28 ae +3, Case (i): WY < oS a,then f(x) = (2? Ae 3) = mat pe 3, Case (i): Wx > 3, then fe) = 42 +3) = [ee Yle-9}) Aja) +3}. fx 2 0, then F(z) 4 +3. ‘This enables us to sketch dhe praph for x > 9. Them we use the fat that fs an even Funetion to refit this part of the graph about the y-axis to obtain the entre graph. Or, we could consider also the eases < — <0. 326 -Land—1 65 CHAPTERT. PRINCIPLES OF PROBLEM SOWING 1 al ve Pa itllz whiae je? 1] : ana? ~ t-2 iajcr itpl<2 So for 0 O and that ja) = —aiFa <0. Ths, de wero dy ify 20 r+bl= nd yl of eee mre yen We will consider ce equation 2 + {= y+ {yin foureases, WeroveO Qe2eyy>0 Case (ij) y>2>0 Cave iti: 2 > Gandy <0 Case (vs < Dandy > 0 Case (ys yee Cae (i rey sd enytenys? reyS1 © yor} 0<2 Crue) wes? 2et —aS2 @ 2b yoebeny se poytetys2d wrene~ys2 eoyorty sd yoenety se peegte soe at CHAPTER PRINCIPLES OF PROBLEM SOLUING Cl a7 ‘Note: Instead of considering cases (iv), (vad oi), we could have ned that the region i mhanged if andy are ceplced by —2 and —3, 0 he region is symmetric about the origin, Therefore, we need only draw cases (i) i), and (Gi, and rotate though 180° about the origin bn eee) = (88) (88) (BB). (a 2 Ha =n(-2+ (or) =in(-2+ Ja ey. EET =) #-(@+))_ (= of 1 ) In —In(x + V3 FT) = —In(x + VaF=T) = -fla) (y= toes VIFF) mentaging sandy weaste = in(y + VT) 3 ea yt VET enya Val & etsy aPt1 = et =f) =e = y= er Ae In(e? 22-2) <0 wade 25 a1 ade <0 > (e-ales <0 > rEl1,3) Since the argument must be positive, Qe-2>0 > [e-(-VS)Jfe- (1+ V]>0 > 2 € (-99,1 = VB) (1+ V3. co). The intrstion oes intra is [2,1 — VI} U2 + VIA 48, Assume that lg 5 raion, Then fog 4 = w/e for natura} mumbers m and n. Changing to exponential form gives us and then raising both ses to the nth povser gives 2” ~ 5°. But 2" is even and 5" is odd. We have arrived at a contradiction, so we conclude that our hypothesis, that Log, 5 i rational, is false, Thus, lo 5 is rational, 15, Let d be the distance travelod on each half ofthe trp, Let ¢ and fa be the times taken for the rst and second halves of the tip For the fist half of the trip we have ¢ 4/30 ad for the second half we have fz d/60, Thus, the average speed forthe total distance __ 2d 2d) 120d _ 1904 entice ip See Bate TB Baad 7 a = A THe erage sped forthe ene ip 37 is 0. mi/h 6 Ley in, g =, and h = 2. Then the le-hand side of the equation is fo (9 +) Infe +2") = sin 2x = sinxeos.r; and the righthand side is Jag + fo ina + Sin = 2sin-x. The two sides are not equal, 0 the given statement i fils. 47, Let S, be the statement that 7" — 1 is divisible by 6. ‘© Siig tue because 7? — 1 = Bis divisible by 6. 1 Assume Si is true, that is, 7* — 1 is divisible by 6, In other wonls, 7 — TO patho L = (6m4 4) T= 1 = 62m for some positive integer mn. Then 42m + 6 = 6(7m + 1), which is divisible by 6, $0 Ses iste. ‘= Therefore, by mathematical induction, 7" ~ 1 is divisible by 6 for every positive integer m, 68 CHAPTER PRINGIPLES OF PROBLEM SOWING 48. Let Su be the statement hat 14:8-4.5-4--- 4 (2n = 1) =? + Si eve Bocause (1) 1] =, Assume Seis tre, that is, 1-+ 8+ 54-4 (2h ~ 3) = 42. the DEBE S44 QEA IE [QkHN) a1 8454-04 BEI) RSD ‘which shows that Sig is tue P+ (2k +2) = (kt DP # Therefore, by mathematia induction, 149-4 54 --> + (2m — 1) =n? for every positive integer n 19, folz) = 2° and fosn(z) = fol Fula) for = 8 1.2, Sale) * fol fol) ~ faz?) = (x2)* = 24, fala) = fol fale)) = fola*) = (a4)? = 28, Jia) ~ fo(F3(2)) ~ fala) = (28)® = 2"... Thus, a general formula is f(z) = 22°" 2. (3) fale) =3/(2—2) and fact = fo fa form =0,1,2, fiey= (25) = floy= (3-9) = ae Qa ie . 9-38 1 ste) = (4 a) * oe Se = Ea Ee nt eras ‘To prove this, we use the Principle of Mathematical Induction, We have alteady verified that J is the for = 2 “Thus, we conjecture thatthe general formulas fan) = Assume thathe forma iste form = thi fo) = ASHER then fen(e)= o2in(e) = alte) = f(a) . 4 2— the De AERO th tn WRF D— es De] RF Be) SHO “This shows thatthe formula for Ju is te fore = k-+ 4. Therefore, 6y mathematica induction, the formula is true For al positive integers (6) From the graph, we can make several cbseratios: The vals at = @ keep increasing as &inereases, 4 The vertical esymptote ges closer to = Las kinereases 1 The horizontal asymptote ges closer toy = 1 ak increases. 4 The a-intercept for fcr i the value of the Vertical asymptote for fi 1 The intercept fr fic che vue ofthe horizontal asymptote fot fir 2 © LIMITS AND DERIVATIVES 24 The Tangent and Velocity Problems 4. (@) Using P(15,250), we construct the following able (b) Lising te values of that correspond tothe poi closest to P(t = 10 and ¢ = 20), we have fa 2 5 | (6,004) BEECH) 55 w | G9,444) 20 | (20,1) 25 | (25,28) (30,0) {6} From the graph, we ean estimate the slope of the tangent tin a P tobe =22 = 933, | RR ‘0 ‘anes ne 2. (a) Slope = 2853282 = YS = 09.67 () Slope = 249=gs0 (0) Blope = igs = 2 = 71 (4) Slope = S284 — 132 = 66 From the data, we see thatthe patient’ heart rate is decreasing from 71 ty 66 heartbeats /minute after 42 minutes. Aer being stable for awhile, the patient's heart ate i dropping, a0 (b) The slope appears be $ “ 8 mre ()y- Fa Ha Mory= deh Dos [Ws casa | osu in] 09 | (oo,oarsess) | a26siss «i | 0.99 | (omn,o4sras7) | o2si236 vy | 0.990 | (0.999,0.198750) | 0.250125 (wv) | 15 (1.5, 0.6) 02 co} 1a } Ga2osassi0) | oases Lor | (.01,oan248) | 0248766 «vi | 1.001 | (.001,0:500250) | 0.289875, 70 CHwBTER2 UNITS AND DERWATIVES () The slope appeats io be () y—1= 3(2 3) oF: 40 = Q mpg @ [25 | @s.o7eri07) —[ossszs6 iy) 29 | (2.9,0.018683) | osigi67 2.09 | (2:99,0.994987) | 0.501256, 2.499 | (2.999,0.900800) | 0.500125 [35 | @sr22q745) | o.4s9100 (wi) | 31 | Ba, 1088809) | 0.488088 (in | 8.01 | (G1, L001988) | 0.498756 (way | 8.001 | (8.001, 1.000500) | 0.499875, @ 5 y= vl) = 40107, At = deh) 92) Gri 2 ~ ( 2.25}: b= 05, %6 ip (2, 2.05}: b= ay 2 05, 25 4 tangent line atx=3 LF Sess 19(2) — 164 [40(2 +f} — 1602+ WY?) ~16 _ 24h — 161? he he _-seeant ine 4x = 2.5 yoved (2)? = 16. The average velocity between times 2 and 2-4 is A 16h, ith £0. i) (2A: = iv) [2,201]: h = 0.01, vane = —24.16 Ys 1, Yyeg = ~25.6 Nfs (8) The instantancous velocity when t ~ 2 (h approaches 0) is 24 /s 6, (@) y= ull) = 10E— 1.8607, Ate ~ 1,4 = 10(4) ~ 1.862)? = 8.24. The average velocity between times Land 1+ is _ y+h) w(t) _ [2OQ+ A) ~ 1861+ 2] — 8.4 Ce) (© (12h f= Lease = 4.42 m/s Gi) (2, La}: = 01, ve = 8.004 m/s (©) [1.1.01 A = 0.001, tyre = 6.27814 m/s 28h — 1.861 a Gi) [1,15]: h = 0.5, tage = 5.35 m/s = 6.28 - L86h, fh 4 0, Gv) [13.011 b = O82, aye = 6264 m/s () The instantaneous velocity when ¢ = I (i approaches 0) is 6:28 m/s 7. (2) (@ Onthe interval], 3), tase = (i) On the inert (2,8), (iy On the interval (iv) On the interval [3,4], 90 = — 33) 3 5 =a) 7-14 T _ s(8)=8(2) _ 107-54 322 a8) =) (4) —9(3) _ 177-107 | a z Tos ‘SECTION21 THE TAYGENT AND VELOcTTY PROBLEMS goatee 48 Fors) = SS 28 For fe} = 2in(n + 2°) aC) a) =) Te 1 fonsse| [=| oasrate yf osnarar os | osausss | {-05 | oars os | ~o1sein o1 Jost | | -01 | o4sozt2 01 | -ogan2r ons | asosao | | 005 | o.onz70 0s | ~o.erai7 aot [ osoert | { -0.01 | oa9ssor oor | -o.0use52 appears that ir, 8,005 | ~9.026407 [0.001 | 2.006007 appears that lim, ein(2 + 2*) =o. SECTION22 THE LIATOFAFUNCTION O75 veya-2 tan 33 1. For f(a) = YEAR 2. For Fr) = 21. For fla) = YEE 12, For (0) = SES = | @ = | 1 [oases } [=r Po2sroa guawor9 a5 [ozs | | -05 | o2sssis sooo oa | ozssist | | 01 | o2sisse soisas 5 | v2ame1 | | 0.05 | 0250780 Doe | aes on 0.599680 aot | o2esis | | aor | o2sors6 aeonoor aaa ‘appears that fim ¥ 025 <4 appene that Him S"3E 9,6 = 2 fig T Wappen that Int Cae OO 3 2-1 yw aa forge) = EL 24 For fo) =P =f we] Le [7 = fe)_| [= Fa) [05 Toossaar| [rs | oasis 0s [riarser] [—os | n2arrir og fomss7 | Jia | oasano 61 Jorima{ | -o1 | osssoss ons | assors6 | | 105 | oss078s 605 | o6isiv6 | | v.05 | ossaaar 6.9 | asians | | or | o.sss022 oor | oso |} oer | osrer0s 0.999 | o.60%200 | | 1.001 | nsesson 001 | a.sssone | | 0.00 | 0.580660 ons te appears that im 2 = 06 = 8 Ie appears that im, 0559. Later we will be able to show that the exact value is ln(9/5) ce since the amerator is negative and the denominator approaches 0 Fiom the postive side as Fe be since the mmeratoris negative and the denominator approaches 0 from the negative sie as 22 > —3~ = 00 since the numerator is positive andthe denominztor approaches 0 through positive values as: — 1 +5 lis, bo oo by (3). 3 fine cot = tin SE co since the numerator is negative and the denominator approaches 0 through positive valves 3. jim rece = ti oo since the numerator is postive and the denominator approaches O through negative values as 2 — Te CHAPTER? UNITS Ano DERWATIVES ap G2, ~ ip HER = tg, Eo = se scene mete te dint ot Fodeyd ot ap approaches 0 through negative values as —» 2 BO fe) = ay = Se = | fe] os ft is [oa From these calculations, it sams that oo | a6 it law Bim {(@) = ~o0and Jim Je) =o. 099 | -337 ao | 320 ogo | 233.7 rom | saxo 0.9909 | 33937 0001 | 883.0 0.99909 | 33,808.7 1.00901 | 3398.3 (by HF slighty smaller than 1, then x — 1 will bea negative nmber close 0, andthe rexiprocal of 2 — 1, thats (2), will be » negative number with large absofute value. So imn_f2 {Wai sight lager than 1, then =* — 1 willbe a smal postive nummer, and is reciprocal, f(2)- will be lage postive umber $0 in, 0} = 0. (6) Teappeass from the graph of that lion fla) = oo and fin, £60 mY a ° : 0 B41 get 3, () The denominator ofy = 3°55 = 5 hy iseqalo zero when ow Oand 2 = 1.5 are = Oand a = 3 (and the numerator is nol), so vertical asyeaptotes of the finetion. - 38. (a) Let (x) = (1-4 a)" ©) 6 Tree 27ist2 “ A 2.71830 2.71828 2.71828 2718e7 271815 271692 Wappeas that ng (1+ 2)!/* = 2.71828, which x approximately In Section 3.6 we will se that the value ofthe limit is exactly e 36, (a) From the graphs it seems that SECTIONZ2 THELOETOFAFUNCTION O77 37, For f(r) = 2” — (2/1000): (b) [7] = | i Ts aor | os 8 dee | -Cometa oases oot | ~tv0an asses) o.0s | oan doses one | “oom 6.008928 0.001 | -9.001000 sions appears that tin f() = 0. 38, For h(a) = BEE. ® 2 ‘age 10 | oss7dorrs 05 | a7oa199 oa | oxrs67209 0.05. | 0.33365700 o.o1 | 033334667 0.005 | 0.33%83667 © = tia] SOT | aaa oa | 4002135 0.001 | 4.000021 20.0004 | 4.000000 te appears that lin f(a) = ~0.002, (0) Iescems thot ry) 0.93833350) 0.33858344 33838000, 0.83833600 083800000 19.00000000 Here the values will vary from one caleulatr to anacher. Every calculator will eventually give fase values. {(@) Asin par (e), when we take small enough viewing rectangle we get incorrect output. es a a we 38, No mater how many times we 00m in toward the origin, the graphs of f(2) ~ in(=/e} appear to consist of altmost-verticat pe Mn Op \ wre + _Braph ofthe tangent function has vertical asymptote a= a =1 < sin < 1, we must havesin.2 + an, Thus, we $+ xn, oroquivalently, sing = 4+ $n, Since Fond son = sein! £ (corresponding to. © 0,90), Jus 95 150° isthe reference angle foe 30°, x — sin * isthe reference angle forsin~! $. Sow = b(n —sin~ are also equations of vertical asymptoes (corresponding tr * £2.24), a2 layity = Vea = 7 From the table and the graph, we guoss chat [0.09 | 5.92531 the fimit of y as «approaches 1 is 6 0.999} 5.99250 9990 | 5.90025 | < 6.5, From the graph we obtain the approximate points of intersection <1 (©) Werneed ta have Ven P(0.9313853, 5.5) and Q(1.0649001, 6.5). Now 1 — 0.913853. 0636 and 1.9549004 — 1 ~ 0.0649, so by requiring that be withia 0.0549 of 1, we ensure that y is within 0.5 oF, SECTION23 CALCULATINGLIMTS USING THE UMIT LANs O78 2.3 Calculating Limits Using the Limit Laws 4.) sin (/(x) + Se] = Bin, Fe) + tm fBoGe)) {Limit Law 1) im, f(@) +5 Jim g(r) (Limit Law 3] = 44 5-2) = 6 6) ti fos? = [im gay]? Limit Law 6) 2 = (©) ing VG) = [Tag TC) thimic Law 113 =a Brey] 3s) 5 9 we TS Tarr eas Bim fe) ee (Limit Law 3} {6) Because he limit ofthe denominator is 0, scant ose Limit Law 5. The given imi, fin, 222, docs not exist bosause he ay" denominator approaches O while the numerator approaches a nonzero number. : ula) h(a vimit Law 5 © tim =a {Limit Law 5] 2) Hi [Limit Law 4) 2. (a) ling (2) + 02) = li (2) + Jingle) = 2+ 02 (©) fa) doesnot xis sino its et-andsghchand ims re nat equal othe given iit das not exit (© si [Foo = ling Fle) ng f2) = 0-1.8-= 0 (8) Sine (=) = 0 and gis inthe denominator, but liza, f(x) = —1 4 0, the piven limit doesnot exis. © lima F(@) = (0 fin VIF = OTTO m9] [tim s(e)] = 2-2 = 16 Te) = VERT =2 80 (CHAPTER? UMNTS AND DERWATIVES. 3 fim (842 41) = Tim Bet 4 im, 25? — im {Limit Laws 1 and2] BI =3(-2)8 42-2)? = (-2) 40) 19.8, and 7 =48484241=50 (227 +1) [Limit Law 5} 12, 1and 3 _ 202) 1 Mra 19,7, and 8) 5 im (14+ YB) (2— 6s" +2) = Jim (1+ YB) > lim (2 Ge? +2) Limit Law 4 = (im 1+ tim) + (tem 26 Hm? + thm 2") (1.2, and 3] = (14 ¥8)- 2-6-8 48%) (7,10,9) = (09) = 390 6. lim (2 + 1)%(t-+3)° (OP him (4+ 8)° {Limit Law 4] [umes a) ames)” 6) = mem a] [tm] a at pape [14 3) = 8-3 19,7, and 8) 1430 1430 . vm (reican) = (neta) iD lim(1+32) 7° - ees] Gl dig bani 143(1) aye aya “ate - =(5) = 5 Send 8 lim, VETTE = fT ETH a 11, 2,and3] 19,8 9nd71 = VIB=8T8 = VIB=4 SECTION23 CALCULATINGLIMITS SNS THELIATLAWS O81 8 lim VIB ¥ Vee ey ay = ia rl (ea ‘10, (a) The fet-hand side ofthe equation is not defined for 2 = 2, but the right-hand side is. [rand 9} (@) Since the equation holds forall x £ 2, it Follows that both sides ofthe equation approach the same limit as. — 2, justas in Example 3. Remember that in finding lim F(2), we never consider = @. " EANE=D megs e2 mm (ete +) pi ten * erie) = Moa a E48 does not exist since x —~2—> Out 2? — a+ 6+ Bas. 2, in 4 ate ate) 2 im le aye = IT ingoy- Wa e-a % be ory ras ~ artes) 46. Br —4-+ Obutz* de> Suse him 222 iin (SH SKA I) — 16 _ my HGH oF mm ¥ Py Seon tec tim te fim DAY) HEE PE far Goede TT 18. By the formula forthe sum oFcubes, we have 242 1 1 1 tte im ee wry Ro esd TAT emo 8 (BEAK OHA) 8 rant on a WP 20 i fg Jim(12-4 0407) = 24+040= 1 (+ VD B- VI) hg ng OT = hing (4 VE) = 84 V5 VUFR=1 VTERH1_ yA mb VIL MR (TEA) 1 1 ig = Maria Fini? 2 CHAPTER? UMTS ANDDERNATIES VeFE-3 VEFB+3_,, _ (@+3)—9 7 Jrete3 {3 Gon ereey) 1 1 Lim a esar3 er +0? tim (r+? =-b=0 @ae=0 A NeTeD rH eed mata) (rye ' 14 ; = lim = Him oe = tig P= SL 2 Ie Tey Ea oT! +n aT IB 1 Tin BRR] ~~ 3G+0) ~ 9 (= VIF Hs VIF) _ at WET Viet) NETS + VTte) = [aa] a= viet Wit mo VIvt(I+viFt) VIrO(L+ viva) 2 (VHD — 5)(VaFTS +5) +O Wate O+5) 2G (4 A)e— 4) “GS D(Ve O15) COVE = Ft) 0.001 | 0.6661668 =o.0001 | 0.6666:67 0.00001 | 0.666817 ' , ‘.oon001 | 0.665662 ‘o.oon001 | e.6666672 o.oo001 | 0.666717 o.o1 | 0.6667167 o.oo1 | 0.6671603 2 ‘The limit appears to be 3 SECTION 23 CALCULATINGLINITS USING THELIMTLAWS C183 (6g ( z GEE) = tm ED a(vT4 +1) fe, (Greet ase) Sasa Ee = him (VIF +1) [Limit Law 3] [VERO BT tv ands (aT ge 0) (0, 3,and7] (F504) rand 2 @) os © = Se) ~.001 | 0.286902 0.0001 | 0.286775, 0.00001 | 0.286754 0.000001 | 0.286752 + 000001 | 0.286752 ‘0.00001 | 0.286749 tim YEEER VS 0 g.00 0.00n1 | 0.2886727 es 6.001 | 0.288511 ‘The limit appears tobe approximately 0.2887, © lim (we4 ee) im —3+2)=3__ 1 I ape) oye) I ty 1 ip ima {Limit Laws Sand 1) 1 Jer (rand 11 1 w= 11.7, ana} z Ex : 38. Let fl2) = —2*, g(r) = 2? cos 20m and h(x) = 2*. Then iff, “1 Scos20ne <1 > We? fla) Sola) VFR < VP TA sinl/s) << VTE M2) < ofa) $ (2). So sine ln, Fe lim h(x) = 0, by the Squeeze Theorem we have fim gfe) = 0 84 CHAPTER? UNSTSAND DERIVATIVES (2? ae 47) = 400) + 35, We ave ing (42-— 9) = 4(4) — 9. Tand Jn 7, Since ds —9.< f{2) NSM Cee Vales Vem < ye, Since tin. ( Y/e) = Dane lip, (Fe) =0.we we ty. [ye] = Ob ae pen Tee 2-8 Pe-820 fa 38. [3] = = -@-3) ife-3<07 (3- ers ees Thos, tim, (22 = Gand = 8)) = fim, (20+ 2 — 3) = Tim (9 = 9) = 318) ~ Jig (e+ fe = 3) = lim (e+ 32 3-43 =6 Since the let and right limits ae equal, im, (2 + |e ~ 31) = 6. e+6 if2+6>0 40, [4 6) = “(246) iPrs6<0 6 fra 6 (46) eco We'll lok atthe ene-sded limits. e+ 12 oe eto as ‘The ffl and right limits are different, son 4 [249 2°] = |o%(2x = 1)| = [29] [2x — 1] <9? x - 1] pen f27! hee 120 far-1 ee z 08 “\a@e~1) itar-1<0 7 \-@r-1) ite cos So [2x °[~(22— 1)] fore < 05. det 2x Thus, tim GL ty PE im 20> [2x9 FF a Sola ae = TY] 0.5 |. Since [x] =~ fore < 0, we have 48, Since [x] = —2 for < 0, wehave lim G -a) 2 96 CHAPTER2 Liu ANDDERWATIVES 49. (9) () x] = 28-8 << —1,50, tm fo] = in (-2) = (i) [5] =-B for << 2.50, Hm fe] = ‘The right and le iis are different, 0 im, [x] does noexist. ii) [2] = -Bfor-B<2< 80 ny fo} = im (3) (©) @ fe] =I form —1 ais not an integer 50. (a) See the graph of y = cos. ; Since —1 < coer <0 on [-x,~n/2}, we have y = f(z) = feosay rN on[-m,~/2). Since 0 < cosa < 1 on |~#/2,0) U(0,7/2}, we have /(e) = 0 on [=n/2.9} 0 (0.2/2). c Since 1 < cost < On (x/2, x], we have J(2) = —1 on (x/2, 1 = Note that (0 (6) (0 im fle) =Oand tim, f(2) = 0,50 tim Fe) Gi As (H/2)", (2) 0,80 tim 2 Gi) As a —> (5/2), fa) + 180 Him, f(z) (iy) Since the answers in pars (i) and (ii) re not equal, lim. f(r) doesnot exis. etna (© Jim, f(e) exits forall in the open interval (~n, x) except e = —7/2and a = x/2. 51. The graph of f(x) = [x] + [=a] is the same as the graph of g(x) = —1 wih Aoles a each integer, since f(a) = 0 for any 41, However, ineger a, Thus, tim f(a) = —L and lim, f(a) = ~1, 50 Jn, Fe) 12)=PI+ 1+ 2+ (-2) = 0,50 im fir) # F12) 453, Since p(x) is polynomial, p(x) = a9 + axe + aa2® +--+ aya", Thus, by he Limit Laws, im (2) = tim (a9 + aa + age? +--+ 42") = ag + ay lim otf 2? + ay Fain + aga? Hoot aye” Xa) ‘Thus, forany polynomial p, ti phx) = pla). SECTION23 CALCULATINGLIMITS USING THELMTLAWS 187 54, Let riz) = ws where pz) and g() ae any polynomials, and suppose that g(a) # 0. Thus, i (a) = tm, ED ED) cigictow sy = 2 ferccie $81. = ro) Jim r(x) mo) [Limiie Law 5] ‘day [Ex 53) “{a). 88. Jim [/(e) — 9 Ste lime —1) = 10.0 =0. Thus, tim fe) = Hay {L/() ~ 8] + 8} = Jim if) — 8] + ims =0+8 =8. Aone: Theale fin, LED=* dosnt asthe ters is apie yO. Wi smpornt ith aia 56. (a) im fe) ©) Jin 58. Let f(x) = [2] and g(x) tout [f(2) + ot)] = 9, Let f(c) = F(x) and 92) = 1 ~ (x), where H is the Heaviside funtion defined in Exercise 1.3.57 Thus, either fog 80 For any value of Then lim, fs) and ie, gf) da notes bat i, [(2)9(2)) = m0 = 0. (2 2)(v5—F +1) 8 —a)(voae +2) 61. Since the denominator approaches 0 as: — ~2, he limit will exist oly if the numerator also approaches as x» 2. Inorder for this to happen, we need 3-2 + a(-2) 4049-0 —2a¢ahS=0 4 a= 15. Wiha 25, the limit hecomes im SEBO MORNE HS) yy MEH) ret ea Deva) Ma ent 82 CHAPTER? UNITS AND DERWATIVES £2, Soluion 1: First, we find the coordinaes of P and Q as functions ofr. Thex we can find the equation of the line determined by these tH points, and thus find the x-intercept (the point 2), and take the limit as r — 0, The coordinates of P are (0,7). “The point is the point of intersection ofthe two cites x? +9? =» and (x1)? +? = 1. Eliminating y fom these equations, we get r? — (eI)? & 8 =142r—1 2 = dr Substituting back int the equation ofthe shrinking cree o find the y-coondinate, we get (Ly)? 4 y= r® (ie pstve yale), Sth coordinates 0 ae ( ry [Now we take the init as x — lim 2 = Sothe limiting postion of Fis the point (4,0). Solution 2: We ada few lines tothe diagram, as shown, Note that ZPQS = 9" (subtended by diameter PS). So LSQR = 99° = ZOQT (subtended by diameter OT), It follows that 4OQS = ZTQR, Also ol ZPSQ = 90" = 25 PQ = ZORP. Since AQOS is isosceles, so is AQPR, implying that QP’ = TR. As the circle C shrinks, the point Q s plainly appevactes the origin, so the point must approach a point twice as fir ftom the origin as that s, the point (4,0), a8 above SECTION24 THEPRECISE DEFNITION OF ALMAT 5) 88 = 2, weneed [a ~ 2| < {2 2] = 4. On the right side, we need ie ~2| < |4P — 2] = 4. Forboth of these conltons to be satisfied at once, we need the more restrictive ofthe two to hold, thats, | ~ 2] < 4. So we ean choose 5 = $,orany smuller postive number 2, On the let side, we need | dy = 4. Also,y=20/ (2? +4) = 0.5 when e = 2,50 = 16:22 Spat. Thm necked ray alee IL 2 ‘numer since this isthe smaller of 5 and 90 Cl CHAPTER LATS AND OERIATIVES 1. Fors = 1, the definition of limit requires that we find such that |(4-+ 3x4) ~2) <1 4 1<44a—309 <3 whenever 0 < lx — 1} < 6 Ifwe plot the graphs of y = 1, y= 4+ 2 ~ 32% and y = 3 on the same sereen, we see that we eed 086 S 2 < 1.11, Sosinee |} ~ 0.86) = 0.14 and J3 ~ 1.31) 1, we choose 5 11 (orany smaller positive suber), Fore = Ot, we sting Ssuch that ((4-+2~ 9) — 2) < 02 go 1.9. 4a Ae? < 21 whenerey 0 < [x1] < 4, From the graph, we se that we need DONS < 1 < 1,012, $0 since (f ~ 0.988) = Gs012 and [1 — 1.912] = 0.012, we choose 5 = 0.012 (or any smaller positive number) forthe inequality t0 hel oe 05< 4, Fors = 05, the definition ofa Simi req whenever 0 < [xr ~ 0) < 6. we plot the graphs of y = 0.5, y 5 on the same sereen, we see that We 59 and |0 ~ 0.76) = 0.75, we choose: need ~1.56 2 < 6.76, So sive j0~ (1.59) 0.70 (or any smaller positive suntan Fire =oa,wemmasnssaanaat[ =! a] coro o9<2aF <1 vteen 0 ) x90 From the graph, we find that y — tan®.r = 20,000 when x & 2.562 and 4000 S81 fore near §. This, we got 56.581 — § 0.010 for \ M = 100%. SECTION24 THEPRECSEDERNTIONOFALIAT Cl ot From the graph, we Find that 28/V=—B=100 = 5.066, ‘Ths, 5-+ 6 5.0659 and 5 = 0.065. tae ers Vf (> 0) 17 842m. ()[A~ 1000) <5 > -3 0) as (b)Fomihe gph 109

9280 0, we need § > O such tha if < |2— 1] < 6, then ya2rt3 [(2r43)~5] [(2r+3) —5] <. Thus lin (22-+3) = 5 by the dfsition of tint 92 Cl CHAPTER? MTS AND OERWATIVES 46, Given e > 0, wenged § > Osueh that iF0 < |x — (~2)] < 6, then » [de +3)-2] O such that PU < Jz — (-B)[ <6 then [Ce ~ de} 13) <2, Bari 42) ~ 1 6, we need 5 > 0 such shat if 0 < |x ~ 4] < 5, hen [C7 =e) ~(-5)) <=. ButeT =e} ~ (See a [sii Qme need 8 > OswehihatifO e-sicte > eM — | <2 By the dentin 29, Given ¢ > 0, Weneed 5 > Osueh that if 0 < [3 Ble 6]ce fe -8l len Bl e-2l [et 3—aj O,wensed b> Onset ifO< Je 15} < 4th | [{3+24)(—20) y ai or <2 [B26 fet lsl [-2][rsisl B-2r-6ice > o 0, we need 5 > Osuch that if0-< [x ~ al < 8, then [x | < =, Sod =e will work, 2A, Given = > 0, we need 5 > O such that iO < [2 ~ al < 8, then je— | < e. But le e| = 0, this will be true no mater what 5 we pick 25, Given ¢ > 0, we need 5 > O such that if 0 < |2—O| |x 0] 0, we need 5 > such that if 0 < Jor ~ Ol < 6, shew [29 ~o] <5? =e. Thus im 2° =O bythe definition of: 21. Given > 0, we need 5 > O such hat if < |2-~ 0) < 4 then |x| ~0] 0, we need 5 > O such that if~5 <2 <9,then|YT=E—o] |YT=Z-0] 0, we need § > Osuch that iO < |r ~ 2} < 5,then |x —4r-4.5)—1) 0, neneed 5 > such basil < J2—8) Bee FACB = |n+d) 0, we need § > Osueh that iO < jx — ( | < 8, then ((22 ~ 1) ~ 3) < © orupon sinpitying we oad JP =a) 0, we need d > O such that 0 < J ~2} < &, then |29 — 8} < ©. Now 2" —8] = |( ~2)(2 +20 44)] ile | < Ly that is, 1 << Byihen 2? 4 Be A < 8? 423} 44 = BG and so Js? ~8| = ln 2) (2? +28 +4) < 19|r—2), Soifwetake 5 = min ft, gf. then O< jx <6 > 29 8] = fr ~2) (2? + 204-4) < fp 19 = e, Thos, by the definition ofa imi, tim 2? 33. Given = > 0, weet = min {2, §}. 10 < |r —3] < S,then|e-3]<2 a -2ee-A [+3) <8 Akol2~ 9} < §, 90 [04 —9] = e+ B\je 3) <8-§ =e. Ts, lim” =9, 34. From the figure, aur choices for 5 are 5: = 3 ~ VTE and x 62 = VOTE 4. The largest posse choice ford is the minimum atop of {5:53} thats, 6 = min{Ss,62} = 82 = VOTE ~8 ere Ore 235, (2) The poims of intersection inthe api are (3, 2.6) and (ee, 9.4) — with 24 5 0.891 and 2p © 1.098. Thus, we can take 5 tobe the oo (b) Solving 2 42+ 1 = 3 + gives us two nonwal complex rots and ne el oo, which is (216.4 ose + 1230 FRET Be?) — 12 (6210+ tose + Vase ae ST) 4. then se) = 1.008 274342 and 5 = n(e) ~ 10.009, which apres with our answer in pat, has, § = 9{e) — 1 SECTION24 THE PRECISE DEFINTION OF ALT. 95 M61 Ginga rs Late> Ode eben. ehnetoaantrd > hh 2) < entra l> is suitable, Thus, we should choose 5 min {1,2} 2 Showing that 5 works Given > Owe let 5 1 n{1,2e} 10 < [x 2) 1 0, we must find 5 > 0 such hee [y— a] < e whenever 0 < jx —al <6. But Ive ~ val = JE=*1- « e(fiam hin). Nowiwe can finda postive conan C sch hat V+ Yi > C en eel ! < EA Fat vaanto C= [To Vasa table choice forthe conta. Sols ~ al < (,/Ja-+ va). This sugges that we et 5 = min {30, (\/Fa+ va)e} 2. Showing shat 5 works Given e > 0, we let ruin (J (Fat vA) 10 < fe al <6 thon le alc da > yE4 a> [Jus yocesinpan 1), Abole —ol < (\/Jo+ va) a0 aya = eal < (V+ vale Main Be < Cae ay 38, Suppose that finn I(t) = L. Given e = 4, thee exists > Osuch hae < | <5 = IM — nh <4 ‘Therefore, lin Y= Vi bythe definition of a imi, LP 4. For-o L< 4. Thiscontadis / > 4. Therefore, lim, H(t) doesnot exis. 58, Suppose hai, 2) = L Given € =f there ents 5 > O such thatO < jel < 6 => (fle) — L| < 4. Take any rational number with 0 < |p] <5. Then f(*) = 0, $00~ L] < 4,80 L-< |b) < 4, Now we any iatona? umber swith 0 < {s] <4. Then f(s) = Ly s0 | ~ b) < 4. Hence, 1b < $ 0 > 3, This coneadias 1 < 4,50 lim, f() doesnot exist 49, First suppose tha ig f(2) = L. Then, given e > 0 theeexits 5 > Osothand < raj <5 > fla) 0<|e-al 0< eal < $30 fle) = b] <<. Hones, lim, f(2) = n He) anh ceca > Yf@)—H Oso that Now suppose lim, (2). Lete > Oe given. Since tim f(a ince lim, f(z) = there exists ba > Osothata nD SM 0, weneodd > U such that < [e+ 3} M. Now he Me ® Sa 1 1 reste o \eysic he somes Then 0 < [e+ 3) <5= fer Oxo that In Osuch hat O-<[e~a) APH I —6 Since a _o(2) =e, there exists a > Osuch that < [eal <2 = la) <1 = gle) > e~1. Lets ethe smatier of and dz. Then (M+1—0) +01 fim (f(2) + 92} = 0. ML Ths, (8) Let AL > Oe given. Since lim g(x) = ¢ > O, there exists 5y > O such that < fx — af <5) = lolz) cf ef Since lim f(2) = oo, there ensts 52 > O such that D < fe — a) < 52 = fla) > 2M fe. Let = min {6,62}. Then < |e—af se) ho) > AEE = af 50 hm Heo = 00. (©) LELN < Obe given Since lim g(x) = e.< b, there exists 51 > Osuch that < |e —al
(o(2)— el <—c/2 > gla} Osuch that 0 < Ja ~ al 2Nje. (Notethate 2NjJo > se) gle) < 2X -$ ~ No tim fle) ale) = oo. 2 25 Continuity 4, From Definition 1 im, f(e) = f(A) 2. The graph of has no hole, jump, or vertical asymptote that number: ~4 (removable, 3. (a) The following ste the numbers at which f is discontinuous and the typeof discontinuity 2 jump), 2 (jump), 4 (infinite), (8) Fis continuows ftom the lefLat —2 since lim _f(2) = f(-2). fis continuous from the right at 2 and 4 since la) = J) aod in, (2) = (is continous om nthe side at A sine (A) isundefined 4. giscontinuows om [—4, ~2), (—2,2), (2,4), (4,6), and (6,8). SECTION25 CONTDITY Dar 5. The graph of'y = J(2) must have a discontinuity at An ust show tat an J) = £9) —L, ne@ cost (4) There ae discontinuities at times t = 1, 2,3, and 4. A person tas s— parking inthe 1ot would want to keep in mind thatthe charge will o jump atthe beginning of each hour ater) 4. (@) Continuous tthe location in question, the temperature changes smoothly as time passes, without any instantaneous jumps fiom one temperature to another. (©) Continuous: the temperature ata specifi rime changes smoothly asthe distance due west from New York City increase, without any instantaneous jumps. (©) iscontowous; a the distance due west from New Yor City increases, the lite above Sea level may jp fom one right o another without pong trough af ofthe intermediate values — at ci for example (@) Discontinuous asthe distance traveled increases, the cst of the ride jumps in small inverts (©) Discootinuous; when the lights ate switched on (ro) the current sudden changes betwee and some wonzero vale, ‘vithout passing through all ofthe intermediate values. This is debatable, though, depending on your definition of eurent, 8, Since f and g are continuous functions, ‘ig (242) ~ gl2}] = 2 ey fo) ~ fim ge) (by Limit Laws 2 and 3] 2413) ~ 9(3) {by continuity of f and g at x = 3) = 2-5 g(3) = 30 ~ gf) Since iis given that li [2/(e) ~ o(2}] = 4, we have 10 ~ g(3) = 450 g(3) = 6. By the definition of continuity Fis continuous at 40. sim J) = im (2* + VI=Z) 4 4% Bi, Jl) lm, fo 238)" (ti, 9-42 i “y= 1+ 2-1)']* = Ca) =81 = s-0. By the dfn o canny, continuous ta 98) CHAPTER? LIMITS AND DERIVATIVES a2 2 [Limit Law 5] 2m + lim 3 4 Tig ein 1,2,and3] 2o+3 — [7and 8] = fla) ‘Thus, fis continuous atx = a for every in (2, 90}; that is, J is continuous on (2, 20) 14 Fora < 3, webave fim g(a) = lim 2VS=B= 2 im YEE [Limit Law 3] = Tim@= a 118) ma F [2] =2VF=a [Vand 8] = gla), s0 g's continuous at 2 — a for every a in (—c0, 3). Also, tim_o(2) = 0 = 4(8)-809icontinsous fom the let a 3. Thus 9 contin on (0,9 15. f(c) ~ In |x ~ 2s discontinuous at 2 since f(2) = In0 is not defined a) = GE) TAY dco a bens i) wrt) vray Befconinaos atx besa in (2) doesnot exist & ite Jit f(a) = Lobat (0) = 0 41,0 f is iseontinuous a0 Bet e—3 trys 2. slo) { , fens fi 0) = 22 S38 hg ENED ey but (8) = 6,30 J is discontimous 3 W. F(x) = FEET is a rational function. So by Theorem 5 (or Theorem 7), F is continuous at every number in its domain, {x]22 452 +640} = (2 | (e+ 3)(2+2) £0} = (2 | x# —% ~2} oF (vo, 3) U(-3,-2) U(-2,00). 122. By Theorem 7, the rot function 3 and the polynomial function 1+ 3 ate continuous on i. By pan # of Fheorem 4 the product (x) = Y (1 + 28) is contiouoas on its domain, 23. By Theorem 5, the polynomials * and 2z ~ 1 are continuous on (20,20). By Theorem 7 he toot function / is ‘ontinsous on 0,36). By Theorem 9, the compasite function vZ—T ts eontinuous on its domsin, |, 00). By pat 1 of Theor 4 the sum R(x) = 2° + VZE=Tis continuous on (3, 24, By Theorem 7, the trigonometric fi tion sin. and the polynomial function + 2 are continuous on B, By pa sof Theorem 4 (2) = 2 iscontious oni donsin (= |e —1) 25, By Theazemn 7, the exponential function &~* and the trigonemeti function eos Zt are continuous on (00,0). By part 4 of Theorem 4, L(t) = e-* cos 2tis continuous on (96,20) 26, By Theorem 5, the polynomial 2 — 1 is continuous on (0,20). By Theorem 7 sin“ is continuous on its domain, [3,4]. By Theorem 9, sin~ (2 ~ 1} is continuous omits domain, whichis fel-1s2?=1<1} = {210 <2" <2} = {2 | [ol < vB} = v2 vI} 100 CHAPTER? LATS AND DERWATES 21 By Theorem 5, he polynomial i ~ 1 is continuous on (20, 20)- By Theorem 7, Ins eantnuous on its domain, (0,00). By Theorem 9, In(¢* ~ 1) is continuous an its domain, which is {el -1>0} = {eo apa ela n (-26,-1) (1,00) 2 By Theorem 7, is continvous on (0,50). By Theorems 7 and 9, e¥* i catinuous on 0,96). Also by Theorems 7and 9, cos(e¥#) is conn on [0,0 22 inciony = 7 dominos ay =D ase te left- and right-hand limits at = O are differen, 30, The function y = 1? is discontinuous at = & + ak, where is any integer. The function y = ln(tan®.) is so discontinuous where tan 2 is 0 that is, at 2 = mk. So y = In(tani®) is discontinuous at 2 = any integer 238, Because * — ais continuous on, the composite function f(x) = e* * is continous on, so sim (2) = 0) = = 4, Because aretan isa continuous function, we ean apply Theorem 8, sain 2) =n (iy EES arctan § = 0.588 oo (1522) oifeed VE ife>l By Theorem 5, since /(2) equals the polynomial x* on (—00, 1). fis continuous on (—00, 1). By Theorem 7, since f() sso={ suas the root fnetion Van (1,00), fis cominuous on (2,00). Ate = 2, tim f(a) = fim a? = Land lim sla} li, J = 1. Ths, lin, fr) existe ad equals Alo, /(2) = VI= 1. Thus, fiseomuous at 2 = We conclude that fis continuous on (20,20). SECTION2S cONTNUTY 101 sing if2 n/a (6/408) f8camimioeson (= n/4) U(6/420). im F(a) = tm sr inj = 1/ VB nce the sine fimetion is comtinvousat/4. Silay, lim, lx) = _ lim. wr = 1/VB by cont of the cosine funtion at tte este 1/4. Thus, lin. f(x) exists and equals 1/V3, which agrees withthe value f(=/4). Therein. Fis soncinuous 7/4, 530 Fis continuous on (00, 0°). 1 rrso ” Wsyap2-2 iors? 0.2) (2-2) ite>2 on {Fs continuons on (~c2,0), (0,2 polynomial on jan and (2,20) sine i cach ofthese intro, Now in_f(a) = tim (1+ 24) = and Tin, f(a) = im (2 ~ 2) = 2,90 is dlsconinuous at 0, Since f(0) = 1 Fs coins fom he et a0. Abo, info) = tim (22) “Bia, (2) = in, Ge ~2)* = 0, and f(2) = 0.50 iecontinaous at 2. The ony mmber at which Fi discontinuous 0 rit iteet mrer-{i iis (2) ayy {Fs continuous on (~2e, 1), (1.3), and (3, 0), where it's a polynomial uM «ational Sno, and «compost of a root inetion with a polynomial 74 a0 50 fis discontinuous at 1 respectively, Now inn fe) = fim (e+1) = Band fi, f(e) = tm, (1/ Since (1) = 2.8 continuous Flom the fe ak 1. AS Tim fl) = Tiny (1/2) = 1/8, and lim, (2) = bm, VE~B = 0 J(B),s0 fis iscontinuosa' but itis concinuus fom the igh a 242 reo wsjafe 0sre1 Qee ifeod ‘Fis continuous on (~20,0) and (1,90) sinee on eack of these intervals itis polynomial: ii coacinous on (0,1) since it is an exponential. Now Nin (2) = and tng, f(2) = Nin e* — 1, 0 fs dacontinuous 20, Since F(0) = 1, fis continvous fom the right st, Also iz) = tim e* = eand in, fe) = thm, 2~ 2) = 1, s0 Fis dicontinvows at 1, Singe f(1) =, fis continuous from the fel at 1 $02 0 GHAPTER2 UNITS ANDDERWATIVES 4, By Theorem 5, each pices of F is continuous on its domain, We need to check ercontinaty at = oo GM _ GM eM _oM, SM since F(R) = = ty Sa cp = i Se = F is continuous at R, Therefore, F isa continuous funetion ofr ‘ ife<2 #2) wor ife>2 A cot i, Fle) = iy, SHE = OM 0 im FO) 72 (cu? +20) = 4e-+ 4 and 4 iscaninuous on (00,2 and (2500). Now tim f(a) 2 bead & 2. Thus or f Um, f(x) = iy, (o® ~ex) =8—2e So fiscontinuous <> de+4—=8 toe continuous on (20,00), € waa eo RI) og etd if ered Qe-ath if 223 it rc? (4 2)e~ 2) _ ia Sin fle) = tim =F = i im (e+2)=242—4 la 243 Jig. f0)~ We must have da b-+3 Ave Hig S() = in lim, f(a) = im, (22-045) = 6-048 Wemthoedea+5=6- ath ora ab—=3 @ ‘Now solve the sytem of equations by adding ~2 mes equation (1) 0 equation 2. Ba 4 4b = ~2 100 4= 3 2a T fora in (1) gives us —26 = —1,50—= 4 as well. Thus, for f t0 be continuous on (90,90), 22+ Dlx 1) _ +e + le-1) ae ua ate etl) forsee tet] . (8) fla) = for #1. The discontinuity is removable and g(r) = 2% + 2? +24 1 agroes with / for x # 1 and is continuous on B. =F 2x _ ate? 2) _ ale 2)(e41) 2 4 z xlr+1) [ore?-+2] forx #2 The discontinuity 6) Fe) is removable and g(x) = 2° + agrees with f fore # 2 and is continuous on B. Je) = im {in 1,50 Jim Jo} does nt Nim 0 Oand tim, f(2) exist. The discontinuity at = = isa jump discontinuity: SECTION2S CONTNUTY 2 103 rrr F does not satisfy the conclusion of the £ does satisty the conclusion ofthe Intermediate Value Theorem. Intermediate Value Theorem. 48. fle) there isa number ein (31,82) such that fe i + 10sin i continuous on the interval [81, 32), (31) = 957, and (32) © 1030. Since 957 < 1000 < 1080, 1000 hy the Intermediate Value Theorem. Note: There i also.a number ¢ in 1) such that J(c) = 1000. 48, Suppose that /(3) < 6. By the Intermediate Value Theorem applied to the continuous function F on the elosed interval [2,3], the fact that f(2) = 8 > 6 and J(3) < 6 implies tat there is # number ein (2,3) such that f(e) = 6. This contradits the fact that dhe only solutions of the equation f(x) = 6 are follows that f(3) > 6. Bus /(3) # 6 because the only solutions of (x) = Garex and 2 ~ 4, Hence, our supposition that f(3) < 6 was incorrect. It 4. Therefore, (3) > 6. and 7 $2) in(1,2) such that f() = O by the Intermediate Value Theorem. Thus, there isa root ofthe equation 2 + a ~3 i conlinuous on the interval [1,2], f(1) and (2) 15, Since ~2 <0 < 15, there is a number ¢ = 3=Dinthe interval (1,2) 48. fl) = YE+2~1iscombinuous onthe interval (0,1), (0) —A,and f(1) ~ 1, Sine 1 < 0 < 1, there is a number cin (0,1) such that f(c) = Oby the Intermediate Value Theorem, Thus, there is aroot ofthe equation YH +r — 3 ra — in the imterval (0 1). 48, (2) ~ cosa ~ wis continous onthe iter 0, /(0} = 1, and f(L) = 6081 ~ 1s» ~0.46, Since ~D.46 < 0 < A, thee is a number c in (0, 1) such teat f(c) = 0 by the Intermediate Value Theorem. Thus, there is a root of the equation cost — 2 =0,0rcose in the interval (0,1), 50. f(z) = Inz ~€-* is continuous on the interval [1, 2}, f(1) =e! = —0:37, and f(2) = m2 0.58. Sinee Oy the lacermedite Value Theorem, Thus, there isa root “0.87 < 0 < 0.56, there is number cin (1,2) such that f(o) of the equation Ine ~¢-* = 0, or In = o~*, ithe interval (2,2). 81. (@) f(a) 10> 0.46, there is @ number cin (0,1) such that f(c) = cos:r — 2 is continuous on the interval [0,1], f(0) > O.and f(L) = cos 1 0.46 < 0, Since by the Intermediate Value Theorem. Thus, thee is a oo! ofthe equation case ~2* = 0, or cos = 2°, inthe interval (0,1) (b) (0.86) 501016 > Oand f(0.87) ~ ~0.014 < 0, so ther is 2 oot betwen 0.80 and 0,87, hati inthe imerval (0.86, 0.87). 82, (a) f(2) = ine —3-4 2 is continuous onthe interval [1, 2}, F(2) = —1 < 0, and (2) = n3 1. <0 < 1.7, there isa number cin (1,2) such that f(c) = O by the Intermediate Value Theorem, Thus, ther isa toa! oF NLT > 0. Since the equation Ine ~ 3-42" ~0, or Ine =~ 2x, inthe interval (1,2) toe Cl CRAPTER2 LIMITS AND DERWATIVES (6) (1.84) = 0.88 < Wand J(1.85) © 0.0001 > 0, so there is a root between 1:34 and 1.5, thas, nthe interval (1.34, 1.35). 58, (a) Let f(a) ~ 100e~*/" — 0.0122, Then J(0) = 100 > Oand (200) = 100e"* — 100 = 4.2 < 0 So by the Intermediate Value Theorem, there i @ number «in (0, 100} such tha Je) = 0 This implies that 10°11 = anne (b) Using the imersect feature of the graphing device, we find chat the cs we root of the equation is 54. (a) Let P(e) = arctane +2 — 1, Then f(0) = 70.347, correct to thee decimal places, 1 0. Soby the Imermediate Value Theorem, there isa y= arcana number cin (0,1) such that f(c) = 0. This implies that as retane= 1, (by Using the meset fest ofthe graphing device, we find that he root ofthe exton is — 1.520, comet othe decimal places. 55, (4) Fs conus at , then by Theor 8 wih a(h) == we have im (a+b) = F (fim (a+ h)) = Fl) (2) Lete > 0. Since jim F(a-+ 4) = Ja, there exits 6 > 0 such that O-< [hj Loh) = Fla <=. S0if0 < fr~ a}
Othe eral (0 — 80-48) contains bth infinitely many rational and infinitely many irrational numbers. Since f(a) ~ Ove 1 there are infinitely many numbers 2 with 0-< 2 ~ al < 8 and |f(2) ~ f(o}| = 1. Thus, Jim fee) # F(a). fat, Him, f(e) des nt even exist SECTION25 CONTNUTY C495 0 ifs rational 60. 9(2) = is continuous at O, To see why, note that — le} < g(x) < [x], s0 by the Squeeze Theorem se ites itrational n gl) = 0~ 9(0). Bat gis continuous nowhere els. Forfa # 0 and 5 > 0, the interval (a ~ 6,4 + 6) contains both infinitely many rational and infinitely many irrational numbers. Since g(a) = O or, there are infinitely many numbers with 0 fea) < 6 and Ja(2) ~ g(a} > fal /2. Ths, fim (2) 2 ofa). 64 thor is such a number, tsaiis the equation 24 +1=2

0. Note aso that f(s 8 pynomia, au us continous. So by the Intermediate Value Theorem here isa number e between —2.and--1 such tat f(e) =0,sothate~ 8 41 atta + =O ale? br 2) + Mee? + Be? — 1) =0. Let pla) denote the left side ofthe last oF zo} ‘equation, Sines pis continuous on [=1, 1), p(—1) = —da < 0, and p(1) = 2b > 0, there exists a ein (~1, 1) such that ‘(c) = Oby the Intermediate Value Theorem. Note that the only root of ether denominator that isin (—1, 1) is (1+ ¥5)/2= r tute) = (8 V5—M)a/2 £0, This, cs not a oot of either denominator, s0 P(e) = => «© = ei8 aot ofthe given equation. 63, f(2) = 2" sin(1/2) is continuous on (26,0) L (0,20) since it isthe product of a polynomial and a composite of a ‘trigonometric function al a rational function. Now since —1 < sin(1/2) < 1, we have ~2* O, lim P(e) = jim x =a = F(a) Fora <0, lim F(a) = lima (—) = a = Fla). Thus, Pi continuous at dats, continuous everywhere (Assume Ut contingous on the interval . Then fora € 1, im (2) = i 2) ~ f(a] by Theorem 8. (Ifa an enpoint of 78th appropriate nese init) Sos continous on 1 ife>0 (6) No, the converse is false Fr example, the function f(x) = { Taree Ba onus tx =, bat f(o}] = Lis continuous on 65. Define u(t) tobe the monk's distance frm the monastery asa fineionoftine, on the frst da, and dined) 0 be his distance from the monastery 38 Retion of tine, on the seeond day. Let J be the distance from the monastery othe top of 2heetountan, From the given information we know that n(0) = 0, u(12) = D, (0) = D and d(12) = 0, Now consider the D and (u ~ d)(12) = D.So by the Intermediate Value Theorem, there must be some time fo between 0 und 12 sueh that (11 d)(to) =O © ulto) ~ alto) function u ~ a which is clearly continuous. We ealeuate that (w ~ 4)(0) = So at time fy after 7:00 AM, the monk will be at the same place on both days. 105 (2 CHAPTER? UNITS AND DERIVATIVES 2,6 Limits at infinity; Horizontal Asymptotes 4. (a) Asie becomes large, the values af f(x) appreach 5. (6) As becomes laze negative, the values of f(z) approach 3. 2. (2) The gral of fmetion ean intersect a “The graph ofa function can intersect a horizontal asymptote ‘vertical asymptote inthe sense that it ean 2a even inferseot its horizontal asymptote an infinite eet but not cross it numberof times / [No horizon asymptote ‘One horizontal asymptote ‘wa hari asymptotes 4G ling F(2) = 00 () fim fa) = 20) im, Fla) fim fle) = (©) tint sl) =2 (0) Verticals 2 = “1,2 = 2 Horizontal: y=, 4G) Jim, gle) =2 () tim gfe) =—2 (6) ing (2) = 06 6 fim ot} (©) tm, gfe) = =o (E) Vertical 2 = ~2, = 0, = 3 Horizontal 8 FO) =0, fQ)=1, 6 Him fl) = 00, Jin Sta) 7 diay fox} = 00, Jim. fle) = 00, din, fe) = 1 im, fle) Hol= 0 i lim, £2) Fisait SECTION2S LAMTS AT INFINITY HORIZONTAL ASHUPTOTES 107 4 im, H(2) = 9, 840) =3 tim fe) = 4, 40, Jim fle) = ~o Jim J) £10) im J) = 96 iy ste) lim, fe) = 00. fim, 0.78125, 4. IF J(2) = 27/2", thn a caleulaorpves (0) = 0, f(1) = 05, $2) = 1, f(8) = 1435, 4) = 1,108 J(6) = 0.5625, J(7) = 0.828125, (8) = 0.25, (0) = 0.158208125, J(20) = 0.097655, f(20) (50) ~ 2.2204 x 10", f(100) = 7.8886 x 10-77, 00038147, te appears Ht fim. (2"/2") <0. 42.) roma graph of f(2) = (1 ~ 2/2)" ina window of 0, 10,00) by 0.0.2), we estimate hat fim J(2) = 0.14 (oto decimal places) © From the table, we estimate that lim f(#) = 0.1853 (to four decimal places) e Se) ~~ 0.135308 0.135833, o.135335 | = fim, Be oe [divide both the numerator and denominator by 2" ots (Ret bn 8)/27 (ths highest power of thatappears in the denominate)} tray?) Tm O55) {Limit Law 5) 3 — Jim (1/2) + 8 ~ Tim FF Jim Ga) ~ im 7 [Limit Laws 1 and 2] 3 li (0/2) +4 Jim, (1/22) - isa [Limit Laws 7 and 3] 3-04 400) em 5 of Section 2. - tee [Theorem 5 of Section 2.5] 108 CHAPTER? UMTS AND OERIVATWES Tee ett Bees 86 tim yf PE PETE fim PEE {Limit Law 3) SVT aah ao? VANS Tp ae a et ’ [divide by 2°) \ Amber aay {Limit Law 5} [FP in GS Oy “im maya) ims Tim (ey (CEémit Laws Vand 2] TF Time) > 8 tim Oe) . Tmo s | Mattows TondT B30) 420) >Re {Theorem 5 of Section 2.5) 15 tin be = ty pM M8 Ser3 ~ Ges aye TOT a) Taos 3 Img 7a) 7 F310) =0 im (1/2) lim (2/2) oop a = tim BEES 50) = fin Goa 7 740) 7° PROMR-9) ABA? = 080-9 a ~ my) 5+ AO) 16 en 2B iy Pe sir tim 2S S VP By ay Gu Eu ai ~ Tim 57) 18. Divide both the numerator and denominator by 2* (the highest power of that curs in the denominator), A Wig 2~ tig Ea ig 204 10) (save DF Ltvoie __pap fae fi m on im, AM Taye 7k T0—0 ~ aye Me ki ‘SECTION26 LIMITS AT INFINITY; HORIZONTAL ASYMPTOTES C109 21, First, multiply the actors in dhe denominator. Then divide both the numerator and denominator by dul 5 aw, aut 45 J TS Bet Su FD Ok (eee VE, Via _ mn VOR sous 2 yy EE et VF en Ry St Vora im FT) Ve Ree co) {since 2° 2%. lim, (VOR E— se) = a ee »f Oat Fa)? — (32)? Vorarys, (022 4.2) — 0" 2 ve = Me, ae “Se ie * 7eaeree 2 8. lim (24 VET) = tim (4 am fe a] y Ea ete) ears 22 = 2 Me pV MTG Vitae Te VITO ‘Nore n diving numerator an denominator by, we used the fc that for < 0.2 = — VF [ER EE) tin VETTE) (VP + VETTE) 2 lim, (Vea — Va Fhe) fim, iS tim 2b az)— +b) jiyy a Hal vie Vipart Vath = Warrant vere Vive >in Qe vibe yin 28, lim, cos:r docs not exist because as inereases cos r does not approach any one valve, but oscillates between Land —1 ‘tt CHAPTER? UMlTs AND DERWATVES 8 jim ESE [divide by the highest power oF xn he denominsor} me Tfet Tint because (ja8+ tr 2) ~+ s0 and (L/e* — Ifa? + 1) ~ Las = 0. 8. Foe > 0 EFT > ViF = Soa: 00, wehave VEFT— 00 hati, in, VETFT 3. tim (2 +24) = tim 2"(2-4.1) factor out the lepest power of} = 90 because 2 ase + 00, Ors mn (a 28) = en 284 2) 2 tim BOBS (24 3/2" vide by the highest power of inthe denen] J ra “eer ears aa? 2 Rit Bag ~+ 00. = a9 boeeuse = Bf + Sfx? + oo amt 5/2? (ne Ve" gy Merah 0-2 MA, Fins ean (2? ~ 24) Jin, my, tan" (2?(1 ~ 2°). tweet ¢ = 22(1— 2°), weknow at # > oo as 2-00, since x? > oo and 1 + =00 So im, tan" (22(1 = 3, Since 1 ¢ car < Land e“* > 0, wehmse <6 Means 0 the numerator and stenominator are Both postive, 80 ling fe) ~ 90. The graph eons ou work cy 2 ast 1) ale a1) _ ED iy) ge 92S) = eee G= MEH) ~ E-Mens) ~ ens He) AT “The graph of 4s the same as the graph OF / withthe exception af a hoe in che 2 FE anee+ staph of fot = 1. By lng division, gla) = ES Asa — oo, glx) ~+ +06, so there is no horizontal asymptote. The denerminaver ——a oF gis zero when 2 ~ Tim o(r) =--s0and tim, g(2) = 09,507 = Bisa vertical asymptote. The graph confirms our Work. 20 2 2 tim 2 tie 2 m 2 s0y = 2s a horizontal asymptote, ig =e Te ~ TTT soy = 2Keahoraonalasynon tm, 225 = 222 <0. =Oieshorrotl anne. Te mint anthe enor) SECTION26 LiMfTS ATINFINITY, HORZONTAL ASYKPTOTES C1 11% whene™ 5 =0 im, aalinty* © = co since the numerator approaches 10 and the denominator approaches 0 through positive values as 2 —» (In 5)*, Similarly, det =o. Thus, = In5 isa vertical asymptove, The graph aatins)~ ‘confirms our work, 45. From the graph, it appears y = 1 isa horizontal asymptote, 0° 45000? si ax? 46005 = 34 (2060/2) SF BoM + 100EF 2000 ~ = SF BOD 4 100 + 2000 ~ A TF TOO) + (100/*) + CRO sso , = EEEETG TH 20y = Bisahorizontal asymptote ‘Tie discrepaney canbe explained by the choice ofthe visting window, Try [-100,000, 100,000] by {1,4 10 get a graph that lends crebiliy to our calculation that y = isa horizontal asymprot w voy 4.00) Wo L From the graph, it appears os int there is only one horizontal asymptote, at y ~ 0, and a vertical asymptote at 2:58 1.7. However, ifwe graph the function with # wider viewing rectangle, we se that in fact there seem to be two horizonial asymptotes: one aty ~ 0.8 and ane at y & ~0.5. So we estimate that a dim, 0s (0000) ~0.r22 an 10000) ~ 0.75, s9 wees tt fn EET J(-1000) = 0.1706 and f(—10.000) = 0.4713, so we oar (0) fin, EET in, VEE ine Va = fore > 0) = YF mori For x <0, wehave Vi -r, 90 when we divide the numerator by x, with © <0, we tH CHAPTER? LIITS AND DERWATIVES 47, Let's look for a rational function. (1) Tim Fe) =0 = degre of mmerator < degree of denominator 2) lin fla) = = there xa factor of: inthe denominator (not just x, since that would produce a sign change at = 0}, and the fanetion is degative nese. ~ 0. 12) Jim Ja) = cond fim. f(a) =—s0 => vertical asympote at = 3 ther is tor of (3) inthe denominator (4) JQ) =0 = Bisan 2-intercept; there is at east one fctor of (x — 2) in the numerator. ‘Contining all of this information and putting in a negative sign to give us the desired let-and right-hand limits gives us ag oso pos Jo 48, Since the function has vertical asymptotes 2 = have factors (2 ~ 1) and (2 ~ 8). Because the horizontal asymplote is y = 1, the degree ofthe numerator must equal the and x = 8, the denominator ofthe raronat Function we are tvoking for ase {egreeof the denominator, and the ratio of the leading coefficients must he 1. One possibilty is f(2) ~@eH 4.9 = fla) = ss at(1 = 29) = 28(1- +a) — 2). The rintereept is 4 J{O) = 0. The x-intercept ae 0, 1, nd 1 [found by solving 4) = 0 for) yy ‘Since 2 > 0 fora #0, f doesn't change sig at ~ 0, The function does change sign atx = ~fandax = f, AS ~» 200, fla) = (1 ~ 0?) approaches —20 because x! + 00 and (1 2") —+ eo. 80, y = f(a] = 2%(x + 2)%(u— 1), The yrintercep is f(0) = 0, The e-inercepts are 0,2, and 1. There re sign changes at © and 1 (add exponents on 2 and 2 ~ 1. ‘There is no sign change at ~Z. Aiso, ffx) —* oo asir — os because alt three factors are large. And f(x) ~ 90 a8 2 —» se because 2° + 00, (+2)? + 00, and (w= 1) ~1 ~0. Note thatthe graph of fat.r = 0 fatens out 2, st a) C+ aPC =a), The wintroept £00) = 5CL)PO)! = 3, ‘The e-inteoepts are 3, ~1, and 1. There is sign change a3, but not at ~1 and ‘When is large positive, 32 is negative and the other factors are positive, 0 yf) 2, Whereis large negative, 3 — 2s positive, so im_ fle) = 90. ‘SECTION2.6. LIMITS AT NFNITY; HORONTAL ASYAPTOTES 15 ay (2) = (a "(a4 4s (0) = 0. The a-inercepts ae 0, tuto at 0, lin, fe (e+ 18x 12 +2). The p-interept 1, Land 2. There isa sign change at ~2, and 1. When 2 is large positive, all the factors are postive, so ‘20. When 2 is large negative, only s+ 2 is negative, so lim ste) 58. (0) Since “1 < sing < 1 forall 2, —+ < BE 1 ae > 0. Ase —100,-1/e and /2 + 030by te See “Theorem, (ina}/2-— 0. Thus, Jim, 2 = 4 (6) From par (@, the horizontal asymptote is y= 1 The function 1y = (sinz)/" crosses the horizntal asymptote whenever sin x ~ 0; that is, at = 1 for every integer, Thus, the graph crosses the asymptote an infinite numberof times: 54, (a) In both viewing rectangles, lin to) iy ote seal J fina | P(e) = fie, Ql) = 7 2-10 ” Jn the larger viewing rectangle, P and Q Pile Plo become less distinguishable aa POO) yg B2 a BY +e . sot = (b) Jim. Gey = J 40) + 310) =1 P and Q have the same end behavior. ‘58, Divide the numerator and the denominator by the highest power of xin Q). (@) lfdeg P < deg Q, then the mimerator ~ 0 but the denominator doesnt. So tin. [P(r)/QCe)] =. (6) Ifdeg P > cog Q, then the numerator — +100 but the denominator doesn't, so lim. (P(2)/QUa)] = os Givyn <0 (odd) ()n-<. (neven) (depending on the ratio of the leading coefficients of P and Q). EE @nso Gin >0 (wold) (iin > 0 (even) From these sketches we soe that wo itfn<0 1 ifm=0 ._} 0 taro ©) ce itm <0, nod be ifn <0, neven {16 CHAPTER? UNITS AID DERIVATIVES 1 itn 1 vaso 7 — ws =20 it n> 0, nodd (ime = foo a> hm 2” a vt co ifm >0, neven 0 itweo ned ot. fin BE MOE = yy Sand oy T1/ve ~ et JT Oye) him WOE a2 Vet Wo = (2i/e) 19-0 ggg Met = 2 oy fee yee 7 a 8 Sree yar > <10W0 f= —101n10-* = 101n10 by bn? + I 7 0.10 64 et ge) = EL anf) = ae) ~ 5 Noe at “Bin. g(2) = and Fim, f(2) =O. We ate interested in finding he evaluat which (2) < 0.05, From the Bop we Bn that = = 4.808, 0 we thoote W = £5 (or any larger mb. ’ 0 €2 Fore 05, ment an hth whiners > Neha] EEE 15 BEET cas ‘We graph the bee pans ofthis inequality n the same sereen, and find thatit holds whenever > 2.82, So-we choose N= 3 ‘SECTION26. LONTS AT NFINTY; HORIZONTAL AsyMPTOTES C47 ET zat (or any larger number), For © = 0.1, we must have 1.9 < << 2.1, and the graphs show tha this holds whenever > 18.9. Sowe choose N = 19 (or any larger number). a as ye2n oo 0 05, we ned io ind such that Ha] 100 whenever > From he raph, we Eo vert ean see that his neguiy hols for > 2500. So we choose N = 2500 (or an ality hols for: > 2500. So we chose N = 2500 or aty a Taner number) ° 0 65. (@) 1/2" < 0.0001 & 2 > 1/0001 = 10000 «> 2>100 (2>0) (le > Oisgiven ten I/a? ce ee wD Ife & 2>1WElaN=1N% 1 1 1 trene>v + 2>b = [L-o 50 lim 47 |e | oe 6. (@) I/yE< 0.0001 + Y#>1/0.0001 = 10" «> 2 > 10" (0) Is > Ofsgiven then yF ce 4 VED Ie oo e>/e2 LAN Lo [tena] =e cosy 20 1, Fe 0 Then >No > 81. Fore <0,|1/x~0| Toke N = -I/e. Tene Dis given,then nce @ <—We 1/e O,we need N'> Osuch hate > Ne 3> M.Nows> Mas 2 > URE, sotake N= YT. Then 2>N= VM 3 > M30 jim ot {18° 9 HAPTER? LI/TS AND DERWATWES 8. Given Af > O,wenced N > Osuchthat s > No" > M.Nowe® > M ce >InAf,souke 1N = max(2,In M), (Phisensures that N’ > 0) Thon e > N= max(1,InM) = ef > max(e,M) > M, v0 fm ef = ce 70. Definition Let J be» function defined on some interval (oc. Then ffx} = —-o0 means that Yor every negative ‘number M there isa corresponding negative nutiher WN’ such that (2) < AY whenever < N. Now we use the defiit to prove that Tim (1-4 2" ~oo, Given a negative number M, we need a negative number N such that 2 < N= Dest CM Nowl tet eM @ Mt ws VITTT. ths, wesake W = YAT—T and tnd that 2 0 there is corresponding postive number N such that |j(x) ~ 1] <= 7, Suppose that lin. f(2) whonever 2 > N.IPE= Afaythens > N OMe Other isa covtesponding > 0 namely 1/N) such that f(1/1) ~ J] < € whenever < 1 < 6. This proves chat slim, (2/8) = 1 Jig f Now suppose that lim, f(x) = Ls Then for every € > O there ia comesponing negative number N such that (Fee) ~ Of <= whenevere < NIE = A/zthene < Ne IN <1fe <0 & 1/N <4 <0, Thus, forevery «> Other isa corresponding 5 > (namely ~1/1¥) such that L7(4/t) ~ L| < € whenever ~6 < ¢ < 0. This proves that lim F(4/t) = L= Yim fee) 27 Derivatives and Rates of Change 4, (a) This ast the lope ofthe fn hough so points: mung = SY — 2 1» = 10) PBST () This isthe imit ofthe slope of the secant line PQ 2s @ approaches Pm 2 The.curve looks more like line asthe viewing rectangle ges smaller 2 Ls La pee pee’ on “1 t oo a sot a 01 3. (a) () Using Definition | with f(x) = 4r — 2? and P(2,3), Lia) = fla) tim = Die=9) my SECTION27 OERNATWES AND RATESOFOWNGE 119 Gi) Using Equation 2 with f(a) = je — 2? and PCL, 3), se fig LOE = HO) ig LOEW =I) yy, MOY =F HP] ~38 in i i sh —3 (b) An equation ofthe tangent tines ~ f(@) ory =2e41 © “ “The graph of y = 2e-+ 1 istangont tothe graph of y = Ae — a the point (1,8). Now zoom in toward the point (1,3) until he parabola ang the tangent linear indistiguishable. 4 s ° 4. (@) (0) Using Definition 1 with f(2) and P10), LY~0 fgg 222" — jpg U2) a1 xt gla) jin 1 Ha(h+))=-12 Gi) Using Equation 2 with f(x) — x9 and PLO}, sm fg LOE N = FEO) yy SOE LO) hg (erm cs ny =o my LENS +SN GPA) ote mab fe =e = fim(-h? = 8h—2) = -2 () An equation of the tangent line isy ~ fla) = (a) a) + y-F0)=S'e-) > 9H ory = 20 +2, © 2 “The proph of y = —22 +2 tangent tothe graph ofy = = — 1° atthe 2 iT) 2 tangent line ave indistinguishable. 5, Using (1) with lx) = Ae~ th - 3— - = Grae ty aS Tangent line: y~2—=—1(e—3) @ y2a—2e9 He yenets ‘a CHAPTER? UNITS AND DBRWATIVES 8. Using (1), act —e~3 (20 2p ~ 3)(2+1) = zal ‘Tangent line: y - 1e-(-) @ ymatd tn MELMT = fig, (VE OVE) BG DWEen 7. Using (1). Tangent line: y~1= 2(r—1) } a Grip Qe 202 & Using Gh m= eno Ihe tT A ete ‘Tangent line: y-0= 22-0) @ y=2r 8. (a) Using (2) with y = f(z) = 3-4 de" — 20°, snow ting LOM) = 0) jg, 3¢AO> HP — 200+ hi)? — (8 4 4a? — 2a") ee fe, i Ae? + Sah + HP) — 20? + Bah + Boh? + 4a? + 20° 7 4a? + 8ah+ 4h? — 2a — Gath ~ Gah? — 2h" ~~ da? + 20° 7 (6) At(1,5): m =8(1) ~6(1)® =2, son equation ofthe tangent ine (@) w isy-5-2@-1) @ y= Be43 At (2,3): m= 8(2) ~ 62)? = (2-2) 8, s0 an equation ofthe tangent Sr +19, 2 a fine is y ~ 10. (a) Using (1), = tim LV vEN Va VE) _ jig, a ws Yar (a= a) (Vat Ve) Janta a(ya> L AQ, tem = s0.an equation ofthe tangent line © iy-1=-He~1) © y=~pe4} [Mt (4, 4): = — yh soa equation ofthe rongent ine bed isy-g=-wle-4) @ oy i ‘SECTION27 DERNATVES ANDRATES OF CHANGE 421 ‘1. (a) The partici is moving to the right when sis increasing: that is, on the intervals (0,2) att (4,6). The particle is moving to he fet when sis decreasing; that i, on the interval (2,8). The particle i standing sill when «is constant; hats, on the imervals (1,2) and (3,4), opie (b) The velocity ofthe particle is equal to she slope ofthe tangent fine of the raph, Note that there is no slope atthe corer points on the graph, On the 3. On the interval (2,3), the slope is ? (cont + 42, (a) Runner A rans ee entive Wsmeter race at the same velocity since the slope ofthe position Fetion is constant. Runner B starts the ace at a slower velocity than runner A, but finishes the race a a aster velocity: (b) The distance between the runners isthe wreates athe ime when the largest vertical! Hine semen fits etwoen the two _vaphs this appears to be somewhere between 9and 10 seconds. () The runners had the same velocity when the slopes oftheir respective posit ‘at about 9.5 s. Note thatthe answers For parts (b) and (c) must be the same for dese wraps Because as soon as the velocity {runner B overtakes the velocity for runner A the distance between the runners starts to decrease functions are equal this also appears tobe 19, Let s(t) = 401 — 602 (2) = toy DAD jpg O10) = 16, =O + AOE BO, 8B = 8842 . ee t-2 g t2 mo +2 8 ig( 2-2) = -8(8) = -24 Thus, the instantaneous velocity when = 2s ~24 f/s. 14) Lee H(d) = 100 — 1.800 HO+h)= HQ) , L0G) ~ 1.86(0 + A}?) — (101.86) I % h jim 10-4 10h = 1.8601 + 2h + A?) — 10+ 1.86 BM ti WOE HO = 1.86 — 72h — 1 86H? — 19-4 1.86 ts te Hs G:28h— 1.860? = vt) ~ fina (6.28 ~ 1. 86h) = 6.28 The velocity of the rack after one second is 6.28 m/s. 122 0 CHAPTER? UNITS AND DERIVATIVES (a+) Ho 7 10a + 10h ~ 1,86(a2 + 20h + f#) 100 + 1.860" 7 sn Let) — 86a +9] = (100 ~ 1.864) sR 100-4 10h — 1.8602 ~ 3.72ah — 1.86%? — 100 + 1.860" _ pe ee ee ee 72a 1.86h) i The velocity ofthe rock when ¢ = « is (10 ~ 8.720) m/s. {iny(t0 8720 ~ 195%) = 10-8.720 (6) The rock will hit he surface when =O 6 10t~ LBGE==0 ¢ H(10—1.861)=0 & f= Oor1. 86 = 10, “The rock hits the surface when ¢ = L0/1.88~ B.A, (4 The veloc orthe rock when cits the sure is v( 2s) = 10 ~3.72(~%) = 10-20 10m/s. ~(a+ ht inp LOSI gy 2 — (0 + Dah + 12) mL haa sR 18. ofa) = im, (20h +h?) Tatas he =Qa+h) fs 2 Bae he 2 32 = Anis and (3) = 32 = 2 m/s. FE ne m/s.v02 pm sand v8) = Fy = mi Sot) 16. (9) The average velocity between times tand t= fs sit th)—slt) _ (CDP -R+ A) IS—(@ SEF IB) _ Pa DG A —Bt—Bh 41 ELS Gaiy-t m 7% ve =(1+h-8) m/s. ( Bak t= 8h .sothe average Gi) [35,4 £= 3.5, h=0.5,s0 he average velocity velocity is2(8) + 1-8 = -L m/s. 182635) +05 -8- 05 m/s (iy (4.5: € = 4,8 = 1, so the avenge veo (i) [5]: €= 4, b= 08, so the average velocity is2(4) +1-8=1 m/s i52(4) 405-8 = 05 m/s 0) o(t) = fim MAN SO _ fin re n—8)= 2-8 sov(4)=0. 17, (0) isthe ony negative value. The slope al x = 4 is smaller than the slope at x = 2 and both are smaller than the slope at =2. Thus, 9'(0) <0 < g'(d) <4'(2) < g'(-2). SECTION 27 OERNATWVES AND RATESOF CHANGE C425 18) Since (5 Using the pontslope form ofa line gives us y~ (3) = A(x ~ 5), ory = 42 — 23. the point (5, ~8) ison the raph of g. Since g'(5) ~ 4 the slope ofthe tangent ine at = 5 is A. (b) Since (4,8) ison y = F(z), f(A) = 8. The slope ofthe tangent tne herween (0,2) and (4,3) is 80 (4) = 4. 18. We bogin by drawing a curve through the origi with a ” ope of to satisfy /(0) = 0 and /"(0) = 3. Since 1 (1) = 0, we will round of our igure so that there is ‘horizontal tangent directly over 2 = make sure tht she curve fas a slope of ~1 as we pass ‘over 2 = 2. Two ofthe many possibilities are shown, Last, we 20. We begin by drawing a curve through the origin with «sfope of Oto satisfy (0) = Oand (0) =0. The curve should have a slope of —1, 8, and 1 as we pass over x= —1,1, and 2, respectively Note: Inthe figure, y” = Owhen 3 “1.27 oF 2.13, 24. Using Definition 2 with f(r) = 32 ~ Suan the point (2,2), we have 1) = my £221 - [22 WP 82H) =? gy (24 HY BE 10h) = =n PETES waa) =7 Son equation ofthe tangent line at (2,2) is y ~2= (x ~ 2) ory = Ta 12. 2. Using Definition 2 with ge) = 1 ~ 2° and the pon (0,2, we have oh) b (0) — gg (= 0-4) v0) %, = fay(-#4) <0 ‘So an equation of the sangene ine is y ~ 1 = OC ~ 0) ory 23, (a) Using Definition 2 with F(x) = 5x/(1 +”) and the point (2,2), we have (b) 4 5248) say = um FOAL FO), THOT HE FQ) = fom = i i 4 =. +10 Bh + 2h? + dh. feceriend “eras h i 2h? — h(—2h - 3) RR ah BY MS APE aR 8) So an equation of the tangent line a (2,2) is y—2 = —3(e—2) or y= ‘28 O GHAPTER2 UMTS AND DERWATIVES 26, (a) Using Definition 2 with G2) = de® — 2°, we have GlO+ h) Cla) _ py Wa by? = (a+ W)8]— (4a? = 0?) I Fe) = Jo ss im tn 202 Sah +N — (a + Bath + Ba? +18) — ta? 0? ie 7 Sah 4 4h? ~ Sa®h— Sah — pi rn lim (8a-+ 4h ~ 3a? — Sah ~ N?) = 8a ~ Ba? At he point (2,8), G"(2) = 16 — 12 = 4, and an equation ofthe » tangent line is y ~ 8 = A(x ~ 2), ory ~ dn, Atthe point (3,9), G3) =u-2 y-9= (2-3), ory = —Be + 18. 3, and an equation ofthe anges ine is 25. Use Definition 2 with F(x) = 8 — 2 + du? 4 Ba Me +h) ala + | = (B— 20+ dn’) 2h 44a? + Sah + ® . Ho+W sa) _ iy, = } = fig ESP fy MOBO — nla 4 804 4h) = 24 8a 5 LOM) Ho) — yy ot HY 510+ h) — (ot ~ 50) a my rs tim, (OLE AaEHS GaPh? + dal +h —5a—5h)— a" — Sa), Mable + 67H? + ah? + ht — 5h Mf D sim (4a? + 60th + ah? + A 8) = dat — 5 2. Use Definition 2 with F(t) = (2 + )/(t+ 3). Aarne 1 ett (rh) (2a42h+ Nlo+3)— a+ Nlash+y) Wa) = Fath) = S{a) 1) = im, Berhad ig Rash ies) my (207 £00 + ah + 0h +048) ~ (20? + 20h +60 Has +3) “i Has hier 3) a 5h an 5 5 SN ath Mass) MIKES 4 aF (ormray aad (sh) 1) ~ fig OEM =Led % (02 4 2ah 412 4 Ifa —2)— (@? + fat h-2) Tas k= 2a = 3 (of 2a? + 20h — dah + oft — 242 + a 2) — (a2 beth 2a +0 +h—2) Taha 2) Mo? do tah—2h=1) _ Hat h= 2a ~ AM = lim, = jim tin AA on? MTs R= 3 dod oF SECTION27 DERIVATWES ANDRATESOF CHANGE 125 28, Use Definition 2 with f(2) = 1/VFF3, L 1 VazB~ VaqnF? Juris? Var? Verh avar2 i B ~vaE RoR Vash VaEN ED = in 2 2) = 19 eee ee o eae Vart+ Vath a) Mihyarht 2 yara(varet varhr a) 4 AEST ETT 8° TaFRT EERE avd VOT RTD) 1 Wart ra JarB) Far BPA ing HOE EL- VOT 1a) = ji 029 = HO) = im, hn (EEE VHT) | (30+ 3h+1)~(a41 fy 5a Shad ies) Bap The Ty VT) 3 8 h Veer She 1s Vaal 2V8a7 - Sige ‘Neve that th anno fo Exercses31~36 ar nol unique. (ene 1 1, By Definition 2 Hin P(). where fl) = 2! anda = 1 (By Deisiton 2, fing CEM" 2 70, whee fin) = (1 +3)"andn =. 22. By Detniion2, Jig MIZE —2 _ (16), where 2) = Panda ~ 16 (By Detition 2, fn AIEEE? 90), whew f(a) = YT Fand a = 0 = 8 38. By Bauation 3, tim, 2 (5), where f(x) = 2 and a = 5. Leb <1 iy, . 34 By Equation 3, fim, PEE — # (5/4), where fx) = tan and cose +A) +1 ak cose A) 2 35, By Definon2, fim, F(x) .where f(x) = eos.z and a = 7. (Or: By Definition 2, Jim = 1'(O), where f(z) = cos +) and a = 0. ee 236. By Equotion 3, fie = P(t)ewhere ft) = 16) = 0) = fin, JOEL) iy WOO 5 1) = 946 1 — 00 48) — 4915 fe i (10 +2504 naa = a0h-— 1228) ~ (1004250 — 1225) tim ATA % BT Lada =1. = in, LBPED — tan 41) =m ‘The speed when f= 5 is [11 = 1 m/s. 128 (CHAPTER? UMTS ANODERWATIVES {52 h)=f18) _ ,, (+A) — 6 +m)]- G5) i - 38. v(5) = f'(5) = a, i 1 Sesh6-+) = (84) - 5 im B5 h) py i 5-25 — 5h? —5 oH? = 26h _ ,,, M=5h— 20) 25 aj mn Seem Menem EY ns ray ca ‘The speed when t = 5is |~22 1.04 m/s. $8. The shetch hows he raph fora oom empeaue of 2 ad eigen — tomperature of 38°. The initial rate of change is preter in magnitude than the rate of change after an hour. ‘isha 40. The slope ofthe tangent (that is, the rate of change of ternperature with respect 18 75 = 168 7 200 totime) até = 1 seems tobe about FN 2. -0.7°F/min , 6 (20 , PEMA PE) _ T= 2 perce P(2001)—~ P(2000) _ 68~ 55, «i {2000, 2001) 3 percent/year "zoo ~ 2000 T sip 1990, 2000), 2(2000) = P1999) _ 55-39 6 escyear «i (1999, 2900; P20) — FCS +52 = 16 percem/yea (b) Using the values from (i) and (ii), we have + SAS = 11. pereent/yeas (6) Estimating 4 as (1990, 40) and 22as (2001, 70), the slope at 2000 is = 40_ 50 Boor 1990 ~ 2 5 percent/ year. Pegon) = Pann) _ 5886— 3501 _ 288 £2. (0 (00, ana P02) = PAO) _ 86 99 2885 1995 tne P(2001) ~ P(2000) _ 4709 — 3501 cations/yeae «i (2000, 2001}; PERE) =F UEEOO) _ ATO E01 1208 lations ono), 242000) — P(1999) _ 9506 — 2135 os ay (199, 2000) OE = RES = 1:00 tocations/year 1306 208 (b) Using the values from (ii) and (iii), we have X 1287 locations / year. 2 ‘SECTION27 OERNATWES aND RATES OF CHANGE <1 127 (6) Estimating A as (1999, 2035) and B as (2001, 4960}, the slope at 2000 is al Iocan) oa 1462.5 locations year wat ¢, ac 6500 6 @ OS $20.25 /unit, jiy MC _ CUOL) = C100) _ 6520.05 = 6500 — 655 05 (0 Ee > $20.05/ nit (ty $1200 +A C100) _ [5000+ 10(100 + 1) + 0.05(100 + W)*| — 6500 _ 20K + Mash? % 7 a = 20+ 005h,h40 CLA00 + 8) — C(A00) So the instantaneous rate of change is lim, = 4 av =vec49) v9 = an (154) onan (1-8) nen | (og AE) —( 2) Canes are) t 30 190,000, = 100.000 (120 + 28 Se (120-4 20+ Dividing AV by 4 and then Jeting h +0, we sce thatthe instantaneous rate of change is #2 (¢ — 60) gal/min, 1 | Flow rate (gal/min) | Water remaining VO fast) ° ~ 3883 360,000 10 27707 69,4404 20} 29005 44,4447 30 1606.5 25,000 ao) -unt aut 50 8855 arm © 6 0 “The magnitude ofthe flow rate is preoes atthe beginning and pradually decreases to 0 45, (2) (2) is tho ate of change ofthe production cost with respect tothe number of ounces of gold praduced. lis units are dollars per ounce. (b) After 800 ounces of gold have been produced the rate a which the production costs increasing is $17/ounee. So the cost of producing the 800th (or 80st) ounce és about $17, (6) Inthe short term, the values of /"(x) will decrease because more efficient use is made of start-up costs as > increases, But eventually (2) might increase due to lage-scale operations. 12h 0 CHAPTER2 LIMITS AND DERWATIVES. 48, (a) J°(5) isthe rate of growth ofthe bacteria population when t = 6 hours. Its units are bacteria per how, (6) With unlimited space and mutiens,f should increase a inereases; so f"(5) < J"(0). Ifthe supply of nusents is limited the growth rate slows down at some point in time, andthe opposite may be tre. 41, T'(10) ithe rate at which the temperature is changing at 10:00 AM. To eximate the value of (10), we will average the _ 108) = 7000) _ 7281 difference quotients obtained using the times ¢ = Sand f= 12. Let A = = 45 and T(2)—T(10) _ 88-81, a” TW) TU) AFB _ 15735 _ yo, oe 3.5. Then T'(10) = jim, HO FLO) 5, A FB aren (6) 4"(8) isthe cate ofchange ofthe quantity of coffee sold with respect to the price per pound when the price is 88 per pound, ‘The units for /"(8) are pounds/(dolars/pound). (©) #'(8) is negative since the quantity of coffee sold will decrease asthe price charged for increases. People are generally less willing to buy a preiuet when its price increases. 49. (a) S1(Z) isthe ra 1 which the oxyBen solubility changes with respect to the water temperature. Its units are (mg/l.}/°C. (b) For T = 16°C, itappeas that the tangent line tothe curve goes through the points (0,14) and (32,6). So s(~ 9h 8 0.28 (mg/L)/*C. This means that as the temperature increases past 16°C, the oxyee solubility is decreasing ata rate of 0.25 (mmg/L)/°C. 50, (a) $°(T) isthe rate of change ofthe maximum susta are (em/sy/°C. ble speed of Coho salmon with respect to the temperature. Ii units (b) For 7 = 15°C, it appears the tangent tine tothe curve goes through the points (10,25) au (20,82). So ss) = = 017 (em/SY°C. This tells us that at T = 15°C, the maximam sustainable speed of Coho salmon is, 2-10 changing at arate oF 0.7 (cmn/s\/°C: In a similar fashion for T = 25) ve can use the poins (20, 35) and (25, 25) to 25-35 obtain 5'(25) = BaF 2 (em/s)/°C, Asit gets warmer tzan 20°C, the maximum sustainable speed decreases pil 1. Since f(2) = esin(1/x) when 2 # Dand f(0) = 0, we have HOH) = HO) _ jig, singtsa) ~0 ey he Sa ‘values —1 and 1 on any interval containg 0. (Compare with Example 4in Section 2.2.) £0) = fing sin(1/), This mit doesnot exist since sin(1/h) takes the 82. Since f(x) = 2° sin(1/2) when 2 4 0 and (0) = 0, we have (0) = fg LOI LO fn, PSII =O i sina). Since 1 Sin <1, weve =IiL 0, as are the Function values wf rap (a=, posi ine from left co right, the slopes ofthe tangents to graph (d) are positive, then O, chen negative, then O, then then 0, then negative again, and the Funetion values in graph IL follow the same patter. Fins or Exercises 4-1: Ft poten onthe rah ol fry harzna targets nthe geo off Look or atiy amas on be raph cf ~ tare wie a dsortniy on th graph of On ay nena vee has tangent with posi (or negate) spe, heap "wl be postive (or reglie} Kt graph fhe fnctnis ines he goph off wie a haz ine 180 CHAPTER? LTS AND DERWATIVES 0. ” 1 fi ir {2 The slopes ofthe tangent ines on the graph of y = P(t) are always, Positive, so the p-values of y = P%t) ae always positive. These values tart ‘ut reatively small and keep increasing reaching a maximum at about 1 = 6. Then the yalues ofy = P(@) decrease and gt close to zero. The graph of P tells us thatthe yeast culture grows most rapidly after 6 hours and then the growth rate declines, 18, 1-appears that there are horizontal tangents on the praph of A for t = 1963 and t = 1071, Thus, there are zeros for chose values of ton the graph of A. The derivative is native forthe years 1963 to 1971. 1950 1960 1970 1880 1950 2000 14, See Figure in Section 3.3. SECTION23 THEDERNATWEASARUNCTION 131 1s, » 6 yen et “The slope at @ appears tobe andthe slope att Ass increases toward 1, J"(2) decreases om very appears be 2.7, Ase decreases, the slope gets Jaege nombers to 1. Ass becomes lage, J'(2) ges loser 00. Since the graphs are so similar, we might loser 100, As 8 pues, (2) = 1/33 or (2) = Yr tess that (2) =o snakes sense 17, (a) By aoonting ia, we estimate that £"(0) = 9. #'{4) =, #'Q) =2. 23 and 12) = 4 (By symmetny. f"(~2) = ~/"(2). $0 (4) = “LSD = -2, and f'(-2 (6) appears that f"(2) is twice the value of x, so we guess that f"(x) = 2, Fe) = . * "= fim (ae + A) = 20 18, (a) By zooming in, we estimate that (0) =0, (4) 0.75, (6) By symmetry, Fw} = (0). So f"{-3) 0.75, SM) 8,2) = and f'(8) = 27. (VBS (=2) = 12, and fC) = 27 © ™ {Since J'(0) = 0, it appears that f° may have the form f(x} = az?, Using /"(2} = 3, we havea = 3, so f'(e) = 82 60) Pe) = fig LEAN IO) yy CWP AY pg EAE Beh?) cfg SEAT REM S y ME Beh H (90? 4 40h 4 12) = 90" 182 CHAPTER? LNETS AND DERWATIVES He +0) $0) _ jig, E@ += 4) 7 6. 'C)= jm, Jim, J i Domain of / = domain of f* SUA) = FE) py [50+ Y= 94e-+ WF] = (5H 98) SHB mF) fim BL Bh 9° — 18 one i 5h — 181k ~ 9h? = fim Sh HRW 0 gg MB— HSE 9) ma 7 1B = [ity (5 - 18t — 9h) Domain of f = domsin of J” =, 22 Po) = uy MEENA LE) gy SLE HWP G2) 47) 8) 4 he? + B+ 1.58? Jima + 1.61) =32 ALBT— 16a? 42-37 _ 4 Beh Lh? A i i Domain of f = domain of f” = B. 2. Fe) = jim LEH = 1) (2+ A) ae +h) +5] ~(e" 0+) " 7 +h ~8e—3h i a? + doh + = jim ra e fim, (Be? + 0h +? 3) = 80? <9 Domain of, omainof /” Le +h) ~ fe) h (ci h+ YETR) ~ (w+ vB) 7 (4 9FTE We ETE) [sy et nen Vere ya) OS aeons : 1 1 s(\ gema) te Te Domain of, 0,00), domain off” = (0,00). Bath + Bch? +h? — 3h SECTION2S THEOERWATWEAS AFUNCTION 433 VER RETT + VBE (TERETE B ~ VTE i Vie iy Ve (1420428) (1420) iy 2 2 4 [Viet veel OVitmrihe vive 2Vith Boman of g = [- 4.00). domain of = 4:00) Beles Ap Le M =H) yg T ¥) 10 10 fot, = Jim ——— a-Si ae) MS ea ae) Domain of f = domain off $)U (hs00) Mise) ae athe ne 2.01) = in EAD =GO) yy CEBET EAT yy ES TIED = ay GO RADE AE AN) — (AP ane eat) ah fy, eek DeeD Mb Tey hy Ie 4 n =i Gene * OF Domain of = doniain 0G’ = (—00,—1)U (1,00) 14 cine t+ h) — oft) vith vi vi-viah vir vith Bg) = fim, RED = Jims EE in (RR SE) fen), -b \ = “bavi vi (vit veth) MS OnVEE RV is ean) MO VEER VIET VET h) “1 -1 “TT ~ Te (eh par h + 608? $ ark? +E) 2 rr as te RS Jing (40° + 6220 da? 499) = aa Domain of f = domain of f” = R. 434 GHAPTER2 UNITS yD DERIVATIVES 30 (a) (8) Note that the third graph in pat (a) has small negative values for its slope, f°; but as —+ 6°. f° = 90, See the graph in part (4). (9.1) = fim ite «tg VEZETAD — VOTE | VOLTET + OE =f Vere +h) + Von = Lem) 6-2) “1 Ya eri ea] Ia =1 = Him wo Je—z—h+ Ve—a 2Ve-a Domain of f = (20,6), domain of 7” 20,6), eth) fee hy Ae + hy) tet 2 21 a) = fm LEED = LE) yy Mote i +20) Seth 4s 62%h? 4 doh 4 ht 4 24 Qh tae 7 fi SEALS A Ah tO _ Art + 2) in, z lim (433 4+ 622h-4 Ak? +A +2) Uiiso = LU UG) _ 89= 8 990; 1 vraag) ~ ZAM) = YUM) 58-61 bsg 995 — 1994 T So we estimate that U” (1994) = 3{(-0.80) + (~0.50)] ~ 0.6 10051996 1907 080-065 ~0385 34 (a) P(t isthe rate at which the percentage of Americans under the age of IX is changing wih espect Lo ie, ls units are pce per year 6/90. Pt) ‘for small values of fh. (Tota (0, eu fg PEEP, LEN = PO (1960) ~ P(1950) _ 35.7 ~ 31.1 eo 185 1 For 1960: We estimate (1960) by using h = 0.46 P*(1950) = AO und f= 10, and then average the two results to obtain final sina, ha -20 (450) RESO = PUNO) _ A= _ gap P(lg70) = P0960) _ 340 35.7 10 > P°(1960) ToT) — 1960 Ti or So we estnate chat P'(1960) ~ (0.46 + (—0.17}} = 0.148, [2 [isan ioe no So] [PW [osoo oss 085-046 035 0000 128 CHAPTER? LIMITS ANDDERWVATIVES © Pu 195 990 1970 1980 1950 2000 F (€) We could get more accurate values for P(t) by obtaining data forthe mid-decade years 1955, 1965, 1975, 1985, and 1995, 35, f isnot differntiable ats = ~4, because the graph has a comer there, and atx because thee ia discontinuity there. 36. Fis no differentiable at xr = D, because there i «discontinuity there, and at a. 3, because the graph has a vertical tangent there 37 F isnot differentiable at there —1, because the graph has a vertical tangent there, and at» = 4, because the groph has a comer 38, F isnot differentiable atx 1, because there is discontinuity there, and at = 2, because the raph has a come tere 38. As we zoom in toward (~1, 0), the curve appears more and more like # —_1—, straight line s0 (2) = 2-+ v/a is iferenable at = 1. But no mater how much we zoom in toward the origin, the curve doesn straighten ‘out—we ean’t eliminate the sharp point (2 cusp). So fis not differentiable ate =O. 40, As we zoom in toward (0,1), the eurve appears more and more Hike @ —— straight line, sof is differentiable at = 0. But 20 matter how much we ‘200m in toward (1,0) 0¢ (—1,0), te eurve doosn't straighten out—we can't liminate the sharp point (a cusp), So fis not differentiable at y = +1. 4.0 = f.b=J',0= J". Wecan soe this because where « has a horizontal tangent, b = O, and where b hss horizontal tangent, ¢ = 0, Ween immediately see that e can be neither J nor /’, since atthe points where cas a horizontal tangent, neither nor bis equal 0. SECTION2§ THEDERUATIEASAFUNCTION © 137 42, Where dhs horizontal tangents, only cis 0, so d! = e. has negative tangents For < 0 and bis the only graph that is negative for 2 < 0, Soc! = b. bas positive tangents 0» B fexcept at.» = 0}, and the only graph tha is positive onthe same fm fib= f" anda domain is a, s0 bf = a, We conclude that d= J,¢ 48, We can immoiately See that is the graph ofthe aeceleraton function, since at che points where a has horizontal tangent, neither © nor b is equal to O. Next, we note tha « = O atthe point where has a horizontal tangent, so b must be the graph of the velocity netion, and hence, Bf = a, We conclude that cis the graph af the postion function, 44, a must be the jek since none ofthe graphs are 0 at its high and low points. «i O where hasa maximum, 0 Y= 650 where has maximum, soe! =. We conclude that is the positon funtion, cis the velocity, bis she acceleration, amt a the jet 6.2) in LEED — yy Mees EAPO te?) det ah 2? 0h M) 4 deme) _ 5, dh Dehn? my 7 PE (+H h r in t ‘We see from the graph that our answers ae reasonable beeause the graph of | ~ 10 fis that ofa linear funetion and the graph of J" is that ofa constany 6 [ \ fanetion, $2) = Sim LAM =F) _ a a h ms 11 in LEA S02) gg BEBE gig FE HH — iy = tim fy Ha = a rates wy > AM Rates By AES ate 1 1 tye tn LEN =P) — yp “ERIE CF) = 42 yg tes? P= jy es i sae ray? 7 ES heer ae eth 1 er mE ‘We see from the graph tha our answers are reasonable because the graph of 4 isthat of an even function and is negative Seal.» #0, and the raph of 4 that oan odd function (negative for 2 < Oand positive for 2: > 0). 138 GHAPTER2 UMTS AND DERWATES 1") Sesh) = He) e+)? — (e+ my] (a? 2) he i Ald + 2h — 30° — Beh — 1) 7 tim LEM SC@) _ 5, [Set b) — Be am h fm, Jim(4 — 62 8h) = 4G g(a) = a) = P(x +h) — f"(a) [4 = 6(x + h)) — (4 ~ Br} Pe) y= Ba, 7h 6 (-6) a (2) = tm LEH HD Ce) £0) = I ‘The graphs are consistent withthe geometric interpretations oF the Aerivatives because J" has zeros where f has a local minimum and a local maximum, #” has a zero where f” has a lea! maximum, anf" isa ‘constant fanetion equal tthe slope off" 0 yt (6) Drawing tangent lige at ¢= 1D onthe graph ofa, « appears to decrease by 10 A/s* over a period of 205. Soat 105, the jer i approximately —10/20 ~= ~0.5(8/s*)/s orf? 48, (a) Note that we have factored 2 — a asthe difference af two cubes inthe third sep. Ha)~ gla) eal? if — g/t 1) = yng ALO) hy) POE In aE TE = lim potas = pa = Ii Soca rare > gars 4 (6) F10) ~ Jy HOF LO) _ pis Tis fneton nese without ours the limit dos na exist and therefore (0) does not exist. ©) Lim LF SECTION23 THEDERNATWRASAFUIICTION 2139 re 4, ate) et aha a8) (0) 0) = fn MOD lin Gm ata ea aT 2 lim ea _ at go's be sarpargia pane = Ba 5 (©) g(x) = 2 is continuous at x = 0 and d) — 2g = 00, This shows tha 3 la ajhas a vetal angen fine a = 0 6 ifr-626 (2-6 itr POO Le 6) fe 6<0 7 lee rece ny LD= LO) iy Bee Sorte igh-hand limits 1 i ty LD=LO «gy BSM=8 - tn SE 1, Sessoms £6) = Jim, L= LP) oes not exst and fis not iterentible at 6 , yore 2-6 a Ache ay owing te reise) = B= wher way of writing re= Be 82, f(2) = J2] isnot continuous at any integer n, 80 F is not differentiable y at mby the contrapostve of Theorein 4, [Fa is not an integer, then fis lo constant on an open interval containing a, so f(a) = 0. Thus, F@) = 0, x notan integer. (b) Since fz) = 2° fore > 0, we have f”(2 a) fornrni={" [See Exercise 2.8.17(d).] Similarly, since fie) = —c* For ot <0, we have f'(2) = 10) = jj LO. 2e for <0, At = 0,wehave ela = fing 222) = tf So J is differentiable at 0. Thus, fs differentiable forall (6) From part (6), we have "(x 2x ifezo ae iPro ‘WoO CHAPTER? Lists NO DERWATIVES = Lt b= Ha} # a) 4) = tin ZEAL (4) = tim S42 —/0) S4(4) = fim, a o continuous at 4. Since f(5) isnot defined, is discontinuous at 5. 0 eco (}s@)=p 5-2 itocacd 1(S—2) 224 ‘These expressions show tha / is continuous on the intervals (90,0), (0,4), (4,5) and (5,09). Since _I(2) doesnot exis, so is discontinuous (and therefore not lim, $2) = fim (5-2) =5 40 him fee), differentiable) at 0 (@) Feom (@), fis not differentiable at 4 since J (4) f (4), and from (e) fis not diferentiable at O or 5, 55 (yy seven then F(a) = tim LCE + A) = f(a) fl-fe-h) = ft-2) Fa pag SES = J iy LEMS) py LAMM pene =n =— Jim £ St2) _ _ pee) hss 8 (b) If f is odd, then a) = Yn LEAD fie ~ hy) flex) Sa) fy * * Sieh fix) = yp HEA et ae =A} f(z) img fle = fla 2 f = jm, fet aase) Therelore, fis even. a) 0 (©) The inital temperature ofthe waters lose to ronm temperature because ofthe water that was in the pipes. When the ‘water from the hot water tank starts coming out, @T/dt is large and positive as T increases tothe temperature ofthe water OMAPTER? REVIEW Ott inthe tank, Inthe next phase, 7'/dt = 0 as the water comes cut at constant, high temperature. Aer some eat becomes small and negative as the contents ofthe hot water tank are exhausted. Finally, when the hot water has run out, AP fd is once again Os the water maintains its (cold) temperature fe) anes a Jn the right wrangle inthe diagram, let Ay he the side opposite angle ¢ and 2x tbe side ancent angle @, Then the slope ofthe tangent ine ¢ is m= Ay/Ax = tand. Note that < & < $. We know (see Exercise 17) thatthe derivative of f(x) =a is /"(2) ~ 22. So the slope of the tangent o the curve at the point (1, 1) is 2, Thus, ¢ is the angle between O and $ whose tangent is 2; that i, db = tan“! 2 6 69? 2 Review CONCEPT CHECK 4. (2) lim f(2) = Fs Soe Definition 22.1 and Figures 1 and 2 in Section 2.2 (6) tim, f() = Le See the paragraph after Definition 2.2.2 and Figure (bin Section 2.2. ©) i fle) jim s(2) (©) fim 1) ‘See Definition 2.2.2 and Figure (a) in Section 2.2. See Definition 2.24 and Figure 12 in Section 2.2. 4: See Definition 2.6.1 and Figure 2 in Section 2.6 2 In general, the imit of fn {ails to exist when the funtion does not approach a fixed number. For each ofthe following functions, the limit fails to exist at fi The left and right-hand “There isan There ae an infinite limits are not equal infinite discontinuity. numberof oscillations 43. (a)-(g) See the statements of Limit Laws 1-6 and 11 in Section 2:3 42 O- GHAPTER? (IMTS AND OERWATIVES 4, See Theorem 3 in Section 23. 5. (a) See Definition 22.6 and Figures 12-14 in Section 2.2, (b) See Definition 2.6.3 and Figures 3 and in Section 2.6 6. (@) y= 24: No symplote (0) y = sinz: No apie (6) y < tana: Vertical asyimptotes x = +7, nan integer (4) y = tan” 2: Horizontal asymptotes y = £5 : 0 (@) y= 1/r: Vocal asymptote x = 0, Ve No meyptte vin asymptte 9 = 0 1. (@) Afni fs continuous ata umber aif 2) approaches f(0 82 approaches ota, tim #2) = J) (6) A fonetion is continuous on the interval (—90, 0) iff is continuous at every real number a, The graph of such a function has no breaks and every vertical line erases it 8. See Theorem 2.5.10, 8 See Definition 2.7.1 40, Sce the paragraph containing Formula 3 in Section 2.7 1, (a) The average rte of change oy with respect tox over the interval [2,2] is Lz) = Lee). Sle) ~ sles) (b) The instantaneous cate of change of y with respect to x at 12, See Definition 2.72. The pages following the definition discuss intemretations of f(a) asthe slope of a tangent line tothe graph of f atx =e and as an instantancous rate of change of f(2) with respect tox when 3 = a 18, See the paragraphs before and afer Example 6 in Section 2.8, 14 (a) A fametion fis terete ata number a ifits derivative” exists © ate otha is, 17 f'(a) exit, (8) Sce Theorem 2.84, This theorem also tells us that if 8 nor ‘continuous at, then f is ot treble at a. 45, Soe the diseussion and Figure 7 on page 159. TRUE-FAISE QUIZ 1. False. Limit Law 2 applies only ifthe individal limits exist these don’, 2 False, Limit Law 5 cannot be applied i the limit ofthe denominator is 0 (it) 3. True. Limit Law 5 applies. 4. Te 5. Fase. 6. False, 1. Tre. 8 False. 9. True 10, False 1 False 12, False 13, True. 4 Tae, HAPTER? REVIEW O14 The limit doesn’t exist since f(}/g(2) doesn’t approach any teal number as 2 approaches 5. (The denominator approaches 0 and the numeracor doesn't) ‘ve know thatthe later limit exists (ands qual 2) conti to} = 6, -].tes a )= Oa) os 80 $(6)g(6) #1. A polynomials continuous everywhere, soli p(x) exis and is egual 19 (6), 1 Consider tim (2) — 923] = lima, ( 3) “This limit is ~oo (not 0), but each of the individual functions approaches oc. ‘See Figute 8 in Section 2.6. Consider f(2) = sinsr for x > 0. Jim, flr) 7 oe and J has no horizontal asymptote. (el) ite dt Consider f(r) {i oe The function f must be continuous in order to use the Intermediate Value Theorem. For example, le 1 ose forall, but Zim f(x) = tim, (2* + 1) See the note ufter Theorem 4 in Section 2.8 Str) exists = Fis differentiable at r => Fiscontinuous at => im f(x) = Fle) vic le HY 5 the srt ample, ify = tic encode wie (22) iets ey ten Yo, (22) = wen 28 oe (82) = f(z) = 2" — 102" + 5 is continuous on the interval [0,2], f(0) = 5. (1) ~ ~A.and f(2) = 989. Sinee = 4.<0.< there isa number cin (0,1) such that f(c) = O by the Intermediate Value Theoreim, Thus, thee is o0t of the equation 2" 102 + 5 = 0 inthe interval (0,1) Simiary, there isa root in (1,2) 144‘ CHAPTER? UMTS AND DERWATWES EXERCISES 4.40) (i) tim, f(@) i) tim, fle) =0 Lily fim, Jf) doesnot ens since the Lf and right imits are not equal. (Thee imi is 2.) (0 fog (0) $2 (0) tiny F(2) = 96 (9) lim _f(@) = 90 (ei) iy la) =4 (i ten) Bandy =. (©) The equations ofthe vertical asymptotes are x = O and 2 (©) The equations ofthe horizontal asymptotes ave y (8) Fis discontinuous at x ~ ~3,0,2,and 4 “The discontinuities are jump, infinite, infinite, and remouebl, respectively, 2 fim slo) = -2 iw, fC} =0, fin, f(2) = 00, lim fla) = 00, ig, (2) = 2, {Fis continuous from the Fight at 3 3: Since the exponential funtion s continous, liye i oo #9 _o 4, Sine ational fnetions are continous, in, =P = ae = 3 bee Das 1 and tin HD nr a) i (08 ama ea Another sofmion: Factor dhe numerator a8 a sum of two cubes and then simplify. hha tim OIE a, (= 1) 1 fh 1? = = 1) 419) eh oe BE n [(h— 0? n+ 3] e B ity ME = oo since (r — 9)! + as — 9and YE > 0 forr 8 ty, ape = oo site (r 9)" + Os and LE > Ofer 9 CHAPTER? REVI Os AE2 = tim, A = sim, 8 Jamal lh Samay A (0 + 1)u+2}u=1) wa Olu 2+ 1)u41) _ 20) a+) TC tn EEO) ~ 8 28a —3)(ve +6 +) _ e46~22 ~(0* 2-6) my == 342) Werte D(verora) NF @-Hlvrro+a) = tim —et2) “ER B(Vertra) 904M 418, Since xis positive, V7 = {a = 2. Thus, 14 Since xis negative, V3 «. Ths, tim tin, MEINE, 4 Gene = AM. ea 8/3) z 15, Lett = sin. Then as e+ a ,sinx 0", sot 0", Thus, tin Lode =e, (a 22"—a8)/at aja! = 22 to Span art ale PPE a ae x Bf ls —8 fio JOP TI-2 VFEEI +s) _ yp, det I~ a? 1 Viet tta| 2% Vitae tte 17. im (VF EIEAT — 2) = tina i - (2-4/5 re VFter> eo [eivide by x = v2? fore > 0] im ete av Jae ei TFOFOT I 2( a), Then a. 00, + 00, and lim We CHAPTER? LOWTS AND OERWATIVES: 21, From the graph of y = (cos®x)/2*, it appears that y = Os orion asymptote and = Dis the vertical asymptote. Now 0 < (cas.r)? <1 => To =O chorizo aso. — oie oer = Lands? = Oa 0, sae OF te vertical asymptote 22. From the graph of y = fle) = Va" Pad - veFAF, vertical asymptots. To obtain a diferent form for f, fs multiply and divide it by its conju hla) = (va? Pett - var az) MEE appears that there are 2 horizontal asympotes and possibly 2 ot} 41) real ett - Veter ae Now Jim, att I Ses en 24.(1/2) since VIF = 2 for Jaa vite teasers ol 2 Ti soy ~ 1 is aorta asymptote. For < 0, we have VF | = —2, so when we divide the denominator by x, With 2 , we need 5 > O such that if0 < [2 — 2| < 6, then (14 — 5x) —4] < e, But \(14—5e)—4| ce [Set IO <2 [-|f-B[ 26, Givene > 0 we must find 5 > 0s that iO < je ~ 0) <6, then | YEO] <=, Now |YF~ Ol =|YE] <= [8-0 =[¥8i= Ya < We == [ol = [YP 0, we need 5 > 0 so that if < Ja ~ 2] < 4 then [22 — de — (-2)| < e, Fist, ote thai ~ 2] < 1 then ceo 2<1s00 2-162 + |e 1] <2. Now lets min (c/2,1}. Then < |e 2<5 > |a® — 82 — (—2)| = ll = 2)(e = 1)] = fe 2|fe— 1) < (€/2)(2) Thus, fim (2* — 32) = ~2by the definition of imi 28, Given Al > 0, wenoed 5 > Osuch that if0 <2 —A.< 6,thon 2;VF—A > M. Thinistwe o> VETT< 2M os 2-4 M. Soby the definition of limit, lim, (2//F=4) = 00. 28. () f(a) = V=Fit's <0, fa) =B— rikO <2 <3, flax) = (23) fe > (0 im, 2) = Jim, (2-2) = 3 (i) tin Fe) = (i) Because of (andi), f(2) does notes.) im. fle) = im (8—2) =O (©) Jim. f(2) = im, 3)? =0 (i) Because f(s) and (0, ln (2) = 0. (9 Fis discontinuous a0 since fn, (2) does not exist. Oo ° fis discontinuous a3 since f(3) does not exist 80. (a) (2) = 22 — FOS 2 <2,9(0) = rif? <2 S8,9(2) = 2-418 <2 A Therefore, lina gf) = Yim (22-22) =Oand tim, g(e) = tm, (2—s) =O. Thus lin fr) = 0= 9(2), se gis continuous at 2 ten g(s) = Bim (2= x)= Land tp, gfe) = Ny (e~ 4) = —2, Ts, Jim ole) = —1 = 9(8).50 ais coninuous a3. lin (2) = fim (@—A) =O and Jim gfe) = in, = = ‘Thus, tim of) doesnot exist, So gi discontinuous at 4. But im. g(x) = % = g(2), 809 is coninsous fom the right at MeO CHAPTER? LIMITS AND DERNATIVES ft ny a 31, str iscomtinuous on B by Theorem 7 in Section 2.5 Since e* fs continuous on Re" is continuous on 3 by Theorem 9 in Scation 2.5. Laslly, 2 i continuous on & since i's a polynomial and the product eis continuous on its domain B by Theorem 4 in Section 2.5. 32. »* — 9 is continuous ow & since its 4 polynomial and / i Continuous on [0 2), so the composition v7 —9 is continuous on {r| 27-92 0} = {8,00). Note tha. ~ 2 40 on this set and so the quotient function gf) is continuous on ts domain, (02, ~3] U 3,00) 38, fe) = 2" 34 +2 80 polynomial soit is continuous on [-2, ~t] and (2) 10.<0<1=f(-1), Soby the Aotemedte Value Theorem there is @ number «in (2,1) such that f(e) = 0 1). is the equation 22° + 2? + 2—Ohos 9 root in (~ 3 f(a) ris continuous on & s0 itis continuous on [0.1]. f(0) =2 >> ife~ 1 (2), So by the Intermediate Value Theorem, there isa number e in (0,1) such that fe) =O. Thus, e“F* <2 = 0, oF €-* =e, has a atin (0,1). 38, (a) The slope ofthe ungont line a (2 2 is f= 12) 2st De +2) a = vost ale +2) = (b) An equation ofthis tangent line is y ~ (= ~8(e~ 2) ory = 82-417, 36, Fora geneva point with 2-coortinate a, we have a) Baye a) 2/(1— 3e) = 2/(1~ 3a) _ ,, 21 = Ba) 200 = 32) FET SoNt ~3eye— a} Tanda a 6 and f(0) = 2, soan equation ofthe tangent line fs y ~ 2 (2 ~0) ory sam equation oe tuner nets — § = Ye +1) ory = Be +S BF. (a) 9= a(t) = 14 28-4 2/4, The average velocity over the time imereal {2,2 + A} LEA) = s(t) _ LRU ALS ENY/A~ 13/4 ohn? wee a os lr a So for the following interval the average velocities ar: AB {L Sl. = 2, vyee = (10-4 2)/4 = 3 m/s (i) (0, 2}, BS Ay tng = (104 1)/4 = 2.75 mys (ii a5: R= stave = (10 + 0.5) /4 = 625 m/s GW) LAs A= .L, Yyee = (10 + 0)/A = 2.525 mm/s CHAPTER? REVIEW C109 (When tenner shy fig SEMA) yy OK 1D asa 38, (a) When V increases from 200 in* to 250 in, we have AV = 250 ~ 200 ~ 50 in*, and since P = 800/V, S00 200 = 5.2 —4 = —0.8 Ib/in?, So the average rate of change ar Sav > 0 (©) Since ” = 800/-P, the instantaneous rate of change of V with respect to P is vip) 800 [P— (P+h)] 800/(P +) — 800/P _ a is MPs AP hich inversely proportional othe squte of P. 99. (0) (2) = Jn HL © 12 hy a2 22) = fy a “4 . ()y-4 = 10(e ~ 2) ory = 100 — 16 = 40. = 64,90 f(x) =a and a 44, (2) #(0) isthe ate at shich the total cost changes with respect tothe intrest rate, Its units are dollas/(percent per year) (b) The total cost of paying of the loan isinereasing by $1200/ {percent per year) as the interest rate reaches 10%. So ifthe meres rate goes up from 10% to 11%, the cost goes up approximately $1200. (6) Asr increases, C increases, So f"(r) will always be postive. @, 150 GHAPTERE LATS ;90 DRRVATVES VF— ae Fh) ~ VS BEA VEX ae Sle +h) + v3 — he i, SOF VE ve women = (3— fa) 9p (Va Be re) +m) aR Seer vais “WE (©) Domain of J: (he radicain must be nonnegative) 3 ~ 52> 0 yp Hohe 46. @ S'(@) Brs3 ve (-o0,8] * Domain of j’: exclude 3 because it makes the denominator zero; £ ne (-00.2) oe 7 (©) Our answer to pat (a) is easonable because f(x) i always negative and f is always decreasing € 48, (9) Ase 400, f(z) = 1, so there is horizontal aa n/B+2) asymptote at y = 1, Ase + 3", f(r) + a0,and as x» — fle) + 20, Thus, there isa vetcal asymptote at» = —3, (6) Note that fis decreasing om (--00,~3) and (~3, 00}, s0 J” is negative on those intervals. AS + -k00, ff 10, Asie + —3" andasae + 3h, f+ 00 darth) doe | = jin FE 2M SO) » SE@ TH SEE _ y,, Bhat (e+ A Aa Be @eny (©) F(@) = fim. = ji he = fy, hBrarhGra) py (UB=Se= 3h + dn ~ 29 — hy) — (2-4 + th = 3233 — he) AGE Ge FIG e a) iy aT igg T Mbt (esi ata) stray ara) Gray (2) The graphing device confirms our graph in part (b) because f isnot continuous, at = —1 because f has a commer, at x = 2became és ot AT, Fis not differentiable: at continuous, and at x = 5 beeavse J bas 2 vertical angen 48. The graph of« has tangent lines with positive slope for 2. < O and negative slope forse > O, and the values of it this pattern, so-enmist be the graph of the derivative afte funtion fora, The graph of has horizontal tangent lines to the le and right of the s-anis and b has 2et0s at these points. Hence, bis the graph of the derivative ofthe Function for e. Therefore, isthe graph of fess the graph of , and bis the graph of J" CHAPTER? REVIEW O15 48. 0"(1000) isthe rate at which the total value of US currency in ctculaion is changing in billions of dollars per yea, To csstimate the value of ”(1990), we will average the difference quotients obtained using the times ¢ = 1985 and # = 1995, sa = 08) CNY ABLE p — £08) = C9) _ 199 PTY ITA _ 9.4, hen (1000) =,p, C= SUMO) 5 AB IOORLPTAB Hoon ary Ou, F*(1985) = =38 = -0.16, 50, (a) Drawing slope triangles, we obtain the following estimates: (1950) ~ 42. and F087) = 9 = 002 (6) The rate of change ofthe average numberof children bor to each woman was increasing by 0.11 in 1950, decreasing by 0.16 in 1965, snd increasing by 0.02 in 1987, () There are many possible reasons: ‘In the baby-boom era (post-WWII, there was optimism about the economy and family size was rising, ‘In the baby-bust ea, there was less economic optimism, and it was considered less socially responsibe to have a large Family « In the baby-boomlet era, there was increased economic optimism and a retum to more conservative atiudes. 1. [flo] < gle) —9l2) S fle) < ala) and tim gi) Him -a(e). “hs, by the Squeeze Theor, tin f(2) ‘52. (a) Note that f is an even function since f(x’) = f(—sc). Now for any integer n, In) + -nJ =n — I++ (0, and for any real number k whichis not an integer, (AL-+(-I6] = 1) ==1.So fim (2) exists and is equal to —1) forall values of a (b) Fis discontinuous at ll integers. PROBLEMS PLUS 4. Lett = sor= 8, Thon Lass so et (ter tei ati 2 jim = tim ait. Imari Iw ojaes cy eee > Pare 5 Another method: Maltply both the numerator and the denominator by (VE-+1)( 9% + Y= +1) JarFh=2 Jawa + ext 2. First ration the mmertor: Jig YEE B=2, VOTES _ jy LES oy since the denominator jig Tareb+o Ma Qarabay {he desominat approaches 0 as: —+ Othe limit wil exist only the numerator also approaches as = —» 0. So we requite that a(0)+b-4=0 = b=4, Sothe equation becomes tim ——2——— = 1 >t > 0) " 2h Jared vive Therefore 3 For} 0,50 [20 — 1] = —(2r—1) and [2 + 1] = 22-41 hse, BESSA yp BED =OAED yy Bi a = Since the slope mor = = = 2a > QaaetH1 => a= he? +4, Ase 0,a— 4, and the limiting position of Qs (0,4) ince [x] <2 < [x] + 1,wehave HL < 2 A fora > 1, Ase 00,2] ~ 00 5. Sine fe] $2 < [a] + Lwetave FF} < 2 < <1+pporee ha se] 0, 1 1 = vo pip and + cb. Ths, tim, 27 = 1 by the Squseze Theorem il TH AS py” These 6. (@) 2}? + fy]? = 1. Since [x]? and Jo]® are positive imtegors or, there are only A cases: Case @: [2] =1,y)=0 415 2<2and0 1sety 2 or-1saty<0 @Forns2 cn fz] =n, Then fa) ty] 1 = fyl=t-n > , 1 —n fim? = tim (wht) a 3 @-a-1n0 [bythe quadratic formula] a = (1 ¥5)/2 = 1.618 or ~0.618, 8 (9) Hew area few possibilities: (&) The “obstacle i the line = y (See diagram), Any intersection ofthe graph of J withthe line y ~ 2 constitutes fixed point, and ithe graph ofthe funetion does not cross the line somewhere in (0, 1) then it must either stata (0,0) {in which cave 0 sa fixed point) or finish at (1, 1) (in which cate 1 isa Red point), {) Consider the fanetion Fe) = f(x) ~ 2, where is any continuous funetion with domain [, 1) and range i (0, 1]. We shall prove that J has fixed pont. Now if f(0) = 0 then we are done: fas a fixed pont (he number), whic is what ‘we are ying 1 prove. So assume F(0) # 0. For the same reason we can assume that f(1) #1. Then F(0) = {(0) > 0 and F(1) = J(1) —1 1. Thus, P only traces out the part ofthe curve with 0. y 0, then Gla + 180°) = (a+ 860°) — Tha + 1800 Also, is continuous since temperature varies continuously. So, by the Intermediate Value Theorem, Chas a 280 on the invrsal[a,a-+ 180") FG(a) < 0, hen similar argument apis (b) Yes. The same argument applies. {6) The sume argument eplies for quantities that vary continuously, suchas barometric pressure, But one cout argue that sltude above sea level is sometimes discontinuous, so the resull might no always hold for that quantity 156 CHAPTER? PROBLEMS PLUS feo) = ji SEA =O) — pg @HMITE += AIC _ » peste xf (2) ict) LEPBANS 5 in ge 8) = 2f' (2) + 0) becouse fi ereniabe and heer continuous. 48, (a) Putz = Dandy =0 in the equation: (0 +0) = 5(0) + 0} +4? 040-0? > 0) = 2/00). Suan (0) from ach side of his equton ges J(0) = 0 = jim, psi) HO) ~ hin, LF(0) +. $10) + Fh + OF =f) a 1a) = tion LELIV= SG) — py UC) + Ho) +h ah] - fe) (9 FG) = fin = Fi, a me r vo [ se +s) “4, We are given that |/(2)| $2? forall. In particular, |/(0)} <0, but jo] 2 90 all, The only vontusion (2) =f) _ |) _ i (2) — $00) tat (0) = 0, Now [FEL O) |-\2)-48 ROE sin wy < = Bt i) ~~ oso y the Suse Tso, tng LE =O) ‘So_by the definition ofa derivative, 4 is itferetiable at 0 and, furthermore, #0) o, 50291 | my = 1.083. ago. | Loam | Sincen99 ¢ 1< 103,27 <0< 28 1.0297 2a) “The function value at = 0 i and the slope at x = Ois 1 (10) =e" han ene cia ae) = spn tintin 0) we ot) = (6) (2) =e grows sore rapidly than 9(2) = 2" when x is loege. 3. F(2) = 186.5 is acopstam fanetion, so its derivative i 0, tha is, f"(x) =O. 4. J(2) = VWisa constam function, so ts derivative is 0, that i, (2) ~ 0. 5s Hos MQ=0-F=-3 6 Fle)= fat > Fe) = Gr") =" 1. J(@) <9? ~de46 > f'(2)=30"~4(1) 40= 3274 BSW = Hse > P= HGP) — 314) 43 312 6 a D=M 48) = SH=Hesay = tare 4o=" 40. h(x) = (2 2)2r+ 3) = 222-6 > hie) =222)~1-0a4e~F Myself get to QyaSe 43 + yf =5le") 40 VG) 24m" = Vie) = 4x(374) WRU) = SE = RED) 18. AQ) VS ay 48. Fla) = Ga)" = (42? 2. f= vin ae Ayratthete = y= darth ByaVite~y) Baya SEMES 8 ogee EQ aC dew adves hao fave a 8 _ ae par 3 Vi Teva eve Devs ‘The ast expression can be written wt -2VvE Byte? > yf =Dsince dz*isa constant. W8 g(u) = VBa + MBH Bus VIVE > yu) = VA() + VI(Hu??) = VE4 27. We irs expand using the Binomial Thotem (see Reference Page 1) HQ) = (et) Sab 8eke + See) S(O Se Bee ES Bia) 3a? +B AE) + (B04) = Be BOE BF 2 hy Senet thet seu? 3 af sae! bo? ~ Dey Bus GHAvB ee $407 me ula dee a(Se?) = Bev 5 1087 or 1/(5 YF) +108

Vous aimerez peut-être aussi